Anda di halaman 1dari 205

BAB 1

BILANGAN REAL

1.1 PENGERTIAN BILANGAN REAL


1.2 MACAM-MACAM BILANGAN REAL
1.3 BILANGAN BERPANGKAT
1.4 SIFAT-SIFAT OPERASI BILANGAN BULAT
1.5 OPERASI PADA BILANGAN BULAT

Pengantar aljabar 1
1. PENGERTIAN BILANGAN REAL
1

Bilangan real adalah sekumpulan bilangan yang terdiri


atas bilangan rasional dan bilangan irasional, atau bilangan
real adalah bilangan yang dapat berkorespodensi satu-satu
dengan sebuah titik pada garis bilangan.

1.1
1. MACAM-MACAM BILANGAN REAL
2

1. Bilangan Asli (A)


Bilangan asli adalah suatu bilangan yang mula-mula
dipakai untuk membilang. Bilangan asli dimulai dari
1,2,3,4,...
A = {1,2,3,4,...}
2. Bilangan Genap (G)
Bilangan genap dirumuskan dengan 2n, n ∈ A
G = {2,4,6,8,...}
3. Bilangan Ganjil (Gj)
Bilangan ganjil dirumuskan dengan 2n -1, n∈ A
Gj = {1,3,5,7,...}
4. Bilangan Prima (P)
Bilangan prima adalah suatu bilanganyang dimulai dari 2
dan hanya dapat dibagi oleh bilngan itu sendiri dan ± 1
P = {2,3,5,7,...}

Pengantar aljabar 2
5. Bilangan Komposit (Km)
Bilangan komposit adalah suatu bilangan yang dapat
dibagi oleh bilangan yang lain
Km = {4,6,8,9,...}
6. Bilangan Cacah (C)
Bilangan Cacah adalah suatu bilangan yang dimulai dari
nol
C = {0,1,2,3,4,...}
7. Bilangan Bulat (B)
Bilangan bulat terdiri dari bilangan bulat negatif, bilangan
nol, dan bilangan bulat positif.
B = {...,-4,-3,-2,-1,0,1,2,3,4,...}
8. Bilangan Pecahan (Pc)
Bilangan pecahan adalah suatu bilangan yang dapat
dinyatakan a/b dalam bentuk , a sebagai pembilang dan b
sebagai penyebut, dengan a dan b ∈B serta b ≠0.
Contoh : 1/2 , 2/5 , 3/7
9. Bilangan Rasional (Q)
Bilangan rasional adalah suatu bilangan yang dapat
dinyatakan dalam bentuk a/b, a dan b ∈B serta b ≠0.
(Gabungan bilangan bulat dengan himpunan bilangan
pecahan)
Contoh: √
10. Bilangan Irasional (I)
Bilangan irasional adalah suatu bilangan yang tidak dapat
dinyatakan dalam bentuk a/b, a dan b ∈B serta b ≠0.
Contoh: √
11. Bilangan Khayal (Kh)

Pengantar aljabar 3
Bilangan khayal adalah suatu bilangan yang hanya bisa di
khayalkan dalam pikiran, tetapi kenyataannya tidak ada.
Contoh: √
12. Bilangan Kompleks (K)
Bilangan Kompleks adalah suatu bilangan yang terdiri
dari bilangan dan khayal.
Contoh: 2 + √

1.1.1 BILANGAN BERPANGKAT


3

n faktor
Contoh : 0 0 0 0 000

Berdasarkan penjelasan diatas maka berlaku rumus-


rumus dibawah ini :
Misalkan ∈ ,
maka :
1.
2.
.
3.
4.
5. ( ) ≠0

6. ( )
7.
8. ( ) ( )

Pengantar aljabar 4
9. √ √
. ⁄ .

1.1.1 SIFAT-SIFAT OPERASI BILANGAN BULAT


4
a. Sifat Komutatif:
a+b=b+a
a.b = b.a
Contoh:
1. 5 + 6 = 6 + 5 = 11
2. 9 . 3 = 3 . 9 = 27

b. Sifat Assosiatif:
(a + b) + c = a + (b + c)
(a . b) . c = a . (b . c)
Contoh:
1. (5 + 2) + 3 = 5 + (2 + 3) = 10
2. (5 x 2) x 3 = 5 x (2 x 3) = 30

c. Sifat Distributif Perkalian Terhadap Penjumlahan


a x (b + c) = ab + ac
Contoh:
5 x (3 + 6) = 5 . 3 + 5 . 6
= 15 + 30= 45

Pengantar aljabar 5
d. Terdapat Dua Elemen Identitas Setiap bilangan a
mempunyai dua elemen identitas, yaitu 1 dan 0,
sehingga memenuhi:
a+0=a
a.1=a

e. Terdapat Elemen Invers Setiap bialngan a mempunyai


balikan atau invers penjumlahan, yaitu –a yang
memenuhi:
a + (-a) = 0
Setiap a ≠ 0 mempunyai balikan perkalian yaitu ,
yang memenuhi: a.

1.1.1 OPERASI PADA BILANGAN BULAT


5
a. Operasi Penjumlahan
a + b = c a, b dan c bilangan bulat
Contoh: 14 + 10 = 24
b. Operasi Pengurangan
a – b = c <==> a + (-b) = c a, b dan c bilangan bulat
Contoh: 10 – (-2) = 10 + 2 = 12
c. Operasi Perkalian
a . b = c a, b dan c bilangan bulat
Contoh: 5 . 4 = 20
(-9) . (-4) = 36

d. Operasi Pembagian

Pengantar aljabar 6
a.
a , b bilangan bulat dan b ≠ 0 c bilangan real
Contoh: .

Pengantar aljabar 7
LATIHAN SOAL

A. Kerjakanlah setiap operasi yang diberikan dan


tentukan sifatnya .
1. 0 0
2.
3.
4. 0 0
5. 20

B. Berilah tanda ketidaksamaan pada pasangan


bilangan real berikut ini :
6. 25 , 30
7. 10, 6
8. ,
9. 7 , -23
10. -3 , -10

C. Urutkan setiap kelompok bilangan real berikut ini


mulai dari yang terendah
11. √
12. √
13. 0 √ 0
D. Urutkan setiap kelompok bilangan real berikut ini
mulai dari yang tertinggi
14. √

Pengantar aljabar 8
15. 8; √

E. Nyatakan bilangan-bilangan berikut ini termasuk


bilangan rasional atau irasional.
16. √
17.
18. 0,333333333
19.
20.

F. Kerjakan operasi yang diberikan.


21. 141 . .
22. 8
23.
24.
25. 135

G. Cari hasil penjumlahan S, hasil pengurangan D, hasil


kali P dan hasil bagi Q dari tiap-tiap pasangan bilangan
berikut ini.
26.
27.
28. -4,
29.
30.

Pengantar aljabar 9
H. Hitunglah.
31. 41.

32. 42.

33. 43. ( )

34. 44.

35. 45.
36. –
37. 0
38.
39. 000 852 . .000
40. .0 .0 .0 — .0 0 .00

I. Hitunglah pernyataan berikut ini, diberikan x = -2 , y


=4 , z =1 , a =-1 , b = 2
46. 3x – 2y +6z
47. 2xy +6az
48. 4b2x3
49.

50.

Pengantar aljabar 10
PEMBAHASAN

1. 0 0

Jadi 0 0 . Sifat Asosiatif


2. 0 0
Jadi . Sifat Komutatif
3. 0 0 0 0.
Jadi . Sifat Asosiatif
4. 10 0
Jadi 0 0 . Sifat Komutatif
5. 20 0 0 0 00 0 0
Sesuai dengan Sifat Distributif .
6. 0
7. 0
8. , dalam desimal 0 sedangkan 0
9.
10. 0
11. √
Urutan dari yang paling terendah : ;
√ ;3
12. √
Urutan dari yang paling terendah
: √
13. √ 0
Urutan dari yang paling terendah : √ 0

Pengantar aljabar 11
14. √ 0 0
Urutan dari yang paling tertinggi : 5 ; 4,2 ; √ ;

15. 8; √ 9
Urutan dari yang paling tertinggi : 9 ; 7,5 ; ; √ ; 8
16. √ merupakan bilangan irasional.
Karena jika diubah kedalam bentuk desimal akan
mengakibatkan desimal tak terbatas dan tak berulang ,
yaitu ..
17. merupakan bilangan rasional.
Karena jika diubah kedalam bentuk desimal akan
mengakibatkan desimal terbatas tak berulang, yaitu 3
18. 0,3333333 merupakan bilangan rasional.
Karena merupakan bilangan desimal berulang.
19. merupakan bilangan irasional.
Karena nilai , yang merupakan
pecahan tak terbatas, tak berulang.
20. merupakan bilangan rasional.
Karena jika diubah kedalam bentuk desimal akan
mengakibatkan desimal terbatas , berulang, yaitu : 0,25
26. 141 . . 34.128 .
=33.329
27. 8 —
28. 0 0
29.
30. 135 0 0

Pengantar aljabar 12
31.
32.
33.
34. 4437..
35. .
36. –
37. 0
38. —
39. 000 852 . .000— 0
40. .0 .0 .0 — .0 0 .00 .0
41. .
( ) ( )
42.

43. ( ) ( )( )

44.

( )
0
45.

-235 ( )
46. 3x – 2y +6z = 3(-2) – 2(4) + 6(1) = - 8
47. 2xy +6az = 2(-2)(4) + 6(-1)(1) = 22
48. 4b2x3 = 4(2)2(-2)3 = -128
Pengantar aljabar 13
49. = = =-
.

50. = =

Pengantar aljabar 14
BAB 2

PERSAMAAN YANG MENGANDUNG VARIABEL

2.1 PERSAMAAN LINEAR DENGAN SATU VARIABEL


2.2 PERSAMAAN LINEAR DENGAN DUA VARIABEL
1. Penyelesaian dengan eliminasi
2. Penyelesaian dengan substitusi
2.3 MODEL MATEMATIKA

Pengantar aljabar 15
2.1.1 PERSAMAAN LINEAR DENGAN SATU VARIABEL
1

Variabel adalah lambang pengganti suatu bilangan


yang belum diketahui nilainya dengan jelas. Variabel
disebut juga peubah. Variabel biasanya dilambangkan
dengan huruf kecil a b c z.

Persamaan linear dengan satu variabel mempunyai


bentuk ax + b = 0 dimana a ≠ 0 dan b adalah konstanta,
penyelesaian persamaan tersebut diberikan oleh x = -

Contoh soal-soal yang dipecahkan

1. Selesaikan tiap-tiap persamaan berikut


a. X + 1 = 5
Jawab : x+1= 5
X=5-1
X=4
Cek: ambil x= 4 dalam persamaan diperoleh
4 + 1 = 5, 5=5
b. 3x – 7= 14
Jawab : 3x – 7 = 14
3x = 14+7
X =
X =7
Cek: 3(7) – 7= 14
21 – 7= 14 , 14 = 14

Pengantar aljabar 16
Persamaan dalam huruf

2. Selesaikan untuk x
a. 2x – 4p = 3x + 2p
Jawab : 2x – 4p = 3x + 2p
2x – 3x = 2p + 4p
-x = 6p
X = -6p
b. ax + a = bx + b
jawab : ax + a = bx + b
ax – bx = b – a
x(a - b) = b – a
x = = -1 diberikan a ≠ b
apabila a=b persamaan adalah identik dan benar
untuk semua harga x

terjemahan kata-kata kedalam simbol-simbol

3. nyatakan tiap-tiap pernyataan dalam bentuk simbol


aljabar
a. suatu bilangan lebih besar satu dari dua kali
bilangan tertentu
jawab : ambil x = suatu bilangan , maka 2x =
dua kali bilangan tersebut, bilangan
yang lebih besar satu dari dua kali
bilangan tersebut adalah 2x + 1

Pengantar aljabar 17
b. suatu bilangan lebih kecil 3 dari 5 kali sebuah
bilangan tertentu
jawab : ambil x = suatu bilangan, maka
bilangan lebih kecil 3 dari 5 kali bilangan
tersebut adalah = 5x – 1

soal bilangan abstrak

4. jumlah dari dua bilangan adalah 21, dan salah satu


bilangan tersebut adalah dua kali bilangan lainnya, carilah
bilangan-bilangan tersebut.
Jawab : ambil x dan 2x sebagai bilangan yang dicari
Maka x + 2x = 21 atau x = 7 dan bilangan yang dicari
adalah x = 7 dan 2x =14
Cek, 7+14 = 21 dan 14 = 2(7)

5. empat kali suatu bilangan tertentu dikurangi 10 adalah 14


tentukan bilangan tersebut.
Jawab :
ambil x = bilangan yang dikehendaki, maka 4x – 10 = 14,
4x = 24 dan
x =6

Soal umur

6. seoran laki-laki berumur 41 tahun dan anaknya berumur


9 tahun, di dalam berapa tahun lagi umur ayah menjadi 3
tahun kali umur anaknya?
Jawab : ambil x= jumlah tahun yang dicari

Pengantar aljabar 18
Umur ayahnya dalam x tahun lagi = 3 (umur anak dalam x
tahun)
41 + x = 3 (9 + x)
41 + x = 27 + 3x
3x - x = 41 – 27
2x = 14
X=7
Jadi umur ayah dan umur anaknya 41+7 = 48 dan 9+7 =
16

1.1
2. PERSAMAAN LINEAR DENGAN DUA VARIABEL
3
Sebuah persamaan dengan dua variabel (yan tidak
diketahui) x dan y adalah berbentuk ax+by=c dimana a,
b, c adalah konstanta dan a, b tidak sama dengan nol.
Apabila kita perhatikan dua persamaan
a1x + b1y = c1
a2x + b2y = c2
maka kita katakan bahwa kita mempunyai dua
persamaan linear simultan dengan dua variabel atau
suatu sistem dua persamaan linear dalam dua variabel .
pasangkan harga x dan y yang memenuhi kedua
persamaan itu dikatakan penyelesaian simultan,
jadi penyelesaian simultan dari x + y = 7 dan x – y = 3
adalah x=5, y=2
sistem dua persamaaan linear dengan dua variabel, dua
metode untuk menyelesaikan suatu sistem persamaan
tersebut diilustrasikan sebagai berikut.

Pengantar aljabar 19
1. Penyelesaian dengan eliminasi

Apabila perlu kalikan persamaan yang di berikan dengan


suatu bilangan sedemikian rupa sehingga membuat
koefisien-koefisien dari salah satu variabel dan kedua
persamaan tersebut sama, apabila tanda koefisien-
koefisien yang sama berbeda, maka jumlahkanlah ,
apabila sama kurangkan
Perhatikan
(1) 2x – y = 4
(2) X + 2y = -3
Untuk mengeliminasikan y, kalikan (1) dengan 2 dan
tambahkan dengan (2) untuk memperoleh

(1) 2x – y = 4 x2 4x – 2y = 8
(2) X + 2y = -3 x1 x + 2y = -3 +
5x =5
x=1
Substitusi x = 1 kedalam (1) diperoleh 2.1 – y = 4
atau y = -2
Jadi penyelesaian simultan dari (1 dan 2) adalah x = 1 , y = -2

2. Penyelesaian dengan substitusi

Carilah harga satu variabel dalam bentuk persamaan dari


salah satu persamaan dan harga ini substitusikan ke dalam
persamaan yang lain.

Pengantar aljabar 20
Contoh :
(1) 2x – y = 4
(2) X + 2y = -3
Dari pernyataan (1) diperoleh 2x – y = 4
Y = 2x – 4

Dan substitusikan y= 2x kedalam (2) diperoleh x + 2y = -3


maka
= x + 2(2x – 4) = -3
= x +4x – 8 = -3
= 5x = 5
x=1
substitusikan x = 1 kedalam (1) 2x – y = 4
2(1) – y = 4
2–y=4
-y = 2 atau y = -2

1.1
2. MODEL MATEMATIKA
3

Merupakan bentuk pertidaksamaan-pertidaksamaan


matematika yang didapat dari persoalan- persoalan
kehidupan sehari-hari
Contoh
Tanah seluas 1000 m2 akan dibangun rumah tipe A dan B
untuk tipe A diperlukan 100m2 dan tipe B 75m2 . jumlah
rumah yang akan dibangun paling banyak 125 unit
tentukan model matematikanya
Jawab : x = ruumah tipe A
Y = rumah tipe B
Pengantar aljabar 21
X + y 125
100 x + 75y 1000
X 0
Y 0

Pengantar aljabar 22
LATIHAN SOAL

1. 3x – 2 = 7
2. Y + 3(y – 4) = 4
3. 4x – 3 = 5 – 2x
4. X – 3 – 2(6 – 2x) = 2(2x – 5)
5. =
6. =
7. (x – 3) (x + 1) = (x – 2) (x + 3)
8. (2x + 1)2 = (x – 1)2+ 3x(x+2)
9.
10. =3
11. 2(x – p) = 3(6p – x) : x
12. 2by – 2a = ay – 4b : y
13. = :x
14. = :x
15. + = :y
16. Umur doni adalah lima belas tahun lebih tua dari umur
saudaranya, dani, enam tahun yang lalu umur doni
enam kali umur dani, tentukan umur mereka sekarang
17. Sepuluh tahun yang lalu umur jodi adalah empat kali
umur nabil, sekarang umur jodi hanya dua kali umur
nabil, tentukan umur mereka sekarang
18. 2x – 3y = 7
3x + y = 5
19. 3x – y = -6

Pengantar aljabar 23
2x + 3y =7
20. 4x + 2y = 5
5x – 3y = -2
21. + =6
= -4
22. + =4
=3
23. 2x – 3y + z = 6
X + 2y + 2z = -6
4x – 5y + 3z = 10
24. 2x – y + 2z = -8
X + 2y -3z = 9
3x – y – 4z = 3
25. 4x +2y – 3z = 1
X – y + 3z = 5
X + 5y – 12z = 6
26. X = y – 2z
2y = x +3z + 1
Z = 2y – 2x -3
27. 3x+4y = 13
2x+5y = 11
28. x2 – 16 = 0
29. 4t2 – 9 = 0
30. Suatu perusahaan bangunan merencanakan untuk
membangun duatipe rumah untuk menampung 540
orang, banyaknya rumah yang akan dibangun tidak lebih
dari 120 unit, daya tampung menurut jenis rumah adalah
4 orang untuk jenis rumah 1, dan 6 orang untuk jenis

Pengantar aljabar 24
rumah 11, apabila besar uang sewa pertahun sebesar Rp.
90.000 untuk rumah jenis 1 dan Rp. 100.000 untuk rumah
jenis 11 maka pemasukan maksimum yang diperoleh
setiap tahun adalah...
31. Jumlah tipe rumah yang sihasilkan pada soal no 29 agar
pemasukannya maksimum adalah..
32. Diketahui model matematika : 2x + y 6 : x + 2y 6 ; x
0 ; y 0 maka nilai maksimum f(x,y) = 5x + 3y adalah
33. Nilai maksimum dari f(x,y) = 3x + 6y yang memenuhi
syarat : 4x + y 20 ; x + y
34. Seseorang bepergian membutuhkan waktu 2 jam lebih
lama ketika pulang dibanding waktu perginya, dengan
kecepatan rata-rata 50 mil/jam pergi dan 45 mil/jam
pulang, tentukan berapa jam yang dibutuhkan untuk
waktu pergi?
35. Nilai maksimum fungsi obyektif f(x,y) = 8x + 6y dengan
syarat :
4x + 2y 60 ; 2x + 4y 48 ; x 0 ; y 0 adalah
36. Nani menginvestasikan 3000 dengan bunga 3 persen dan
1000 dengan bunga 4 persen. Berapakah yang harus
diinvestasikan jika dengan bunga 6 persen pertahun
pendapatan tahunannya menjadi 5 persen dari seluruh
investasinya?
37. Seseorang bepergian memerlukan waktu total 12 jam
pergi-pulang, dengan kecepatan rata-rata 20 mil/jam
pergi dan 30 mil/jam pulang. Tentukan berapa waktu
yang dibutuhkan saat pergi?
38. Nilai maksimum dari p = 30x +10y dengan syarat ; 2x +
2y 4 ; 6x + 4y 36 ; x 0 ; y 0 adalah

Pengantar aljabar 25
39. Jika p = x+y dan Q = 5x+y maka nilai maksimum p dan q
pada sistem pertidaksamaan :
X + 2y 12 ; 2x + y 12 ; x 0 ; y 0 adalah
40. Menjelang hari raya idul adha , pak mahmud hendak
berjualan sapi dan kerbau, harga seekor sapi dan kerbau
di jawa barat berturut-turut Rp. 9.000.000 dan Rp.
8.000.000, modal yang ia miliki Rp. 144.000.000 , pak
mahmud menjual sapi dan kerbau di jakarta dengan harga
berturut-turut Rp. 10.300.000 dan Rp. 9.200.000 kandang
ia miliki hanya mampu menampung tidak lebih dari 17
ekor tentukan keuntungan pak mahmud dari hasil
penjualan sapi dan kerbau
41. a. Jumlah Umur Ayah dan Umur Ibu adalah 90 tahun
b. Umur ayah : umur ibu = 8 :7
Berapa tahun umur ayah?
42. Seorang dokter menyarankan pasiennya untuk setiap
harinya memakan paling sedikit 10 unit vitamin B1 dan
paling sedikit 10 unit vitamin B2 pasien tersebut
mengetahui bahwa obat yang berbentuk kapsul
mengandung 1 unit vitamin B1 dan 3 unit vitamin B2 dan
obat yang berbentuk tablet mengandung 6 unit vitamin B1
dan 3 unit vitamin B2 , harga setiap tablet 500 dan setiap
kapsul 1000 tentukan biaya maksimum setiap harinya
43. Seorang petani memerlukan paling sedikit 48 unit zat A
dan 42 unit zat B, kedua zat itu didapat dari pupuk i dan
pupuk ii, satu kg pupuk i memuat 5 unit zat A dan 3 unit
zat B, sedangkan satu kg pupuk ii memuat 3 kg unit zat A
dan 4 kg unit zat B, harga per kg pupuk 1 adalah 30000
dan harga per kg pupuk ii adalah Rp. 25.000 , maka biaya
pemupukan minimal sebesar
Pengantar aljabar 26
44. Seorang anak diharuskan minum dua jenis tablet setiap
hari. Tablet jenis I mengandung 5 unit vitamin A dan 3
unit vitamin B. Tablet jenis II mengandung 10 unit vitamin
A dan 1 unit vitamin B. Dalam 1 hari anak tersebut
memerlukan 25 unit vitamin A dan 5 unit vitamin B. Jika
harga tablet 1 Rp. 4000,00 per biji dan tablet II Rp.
8.000,00 per biji, pengeluaran minuman untuk pembelian
tablet per hari adalah .
45. Kelas wirausaha suatu SMK membuat kue jenis A dan kue
jenis B , tiap kue jenis A memerlukan 300 gram terigu dan
40 gram mentega, sedangkan kue B memerlukan 400
gram tepung terigu dan 100 gram mentega, persediaan
yang mereka miliki adalah 5kg terigu dan 2 kg mentega,
jika x menyatakan banyaknya kue jenis A dan y
menyatakan banyaknya kue jenis B, maka model
matematika yang memenuhi adalah
46. Diketahui keliling sebuah persegi panjang adalah 60 cm.
jika panjang dan lebarnya memiliki selisih 6 cm, buatlah
model matematikanya !
47. Pada suatu pertandingan, tiket-tiket dijual dengan harga
3, 5 dan 7,5 . jumlah tiket 5 adalah 3 kali jumlah tiket
3 dan 10 lebih sedikit dari pada jumlah tiket 7,5 .
penjualan totalnya adalah 885. Berapa banyakkah
masing-masing tiket yang terjual ?
48. Mungkinkah kita menukar uang 10 dengan jumlah yang
sama dariuang setengah dollar, quarter dan dine?
49. Harga 3 mangkuk bakso dan 3 gelas es the Rp. 15.000,00
dan 4 mangkuk bakso dan 3 gelas es teh harganya Rp.
19.000,00. Tentukan harga 1 mangkuk bakso dan 1 gelas
es teh?
Pengantar aljabar 27
50. Pada sebuah perjalanan harga tiket adalah 5 untuk orang
dewasa dan 2,5 untuk anak-anak, jumlah penumpeng
adalah 30 orang, dan total biaya yang terkumpul adalah
122,5 . berapakah banyak orang dewasa dan anak –anak
dalam perjalanan tersebuut?

Pengantar aljabar 28
PEMBAHASAN

1. 3x – 2 = 7
3x = 9
X=3
2. y + 3y – 12 = 4
4y – 16 = 0
4y = 16
Y =4
3. 4x – 3 – 5 + 2x = 0
6x – 8= 0
X = =
4. x – 3 – 12 + 4x = 4x – 10
x – 3 – 12 + 4x – 4x +10 = 0
x–5=0
x =5
5.
4t – 18 = 9t +12
4t – 18 – 9t – 12 = 0
-5t – 30 = 0
-5t = 30
t = -6
6. =
2x2 + 5x + 3= 2x2 – 6x + 4
2x2 + 5x + 3 – 2x2 + 6x – 4 = 0
11x – 1 = 0
X=

Pengantar aljabar 29
7. (x – 3) (x + 1) = (x – 2) (x + 3)
X2 – 2x – 3 = x2 + x – 6
X2 – 2x – 3 – x2 – x + 6 = 0
-3x + 3 = 0 X =1
8. (2x + 1) = (x – 1) + 3x(x+2)
2 2

4x2 + 4x +1 = x2 – 2x + 1 + 3x2 +6x


4x2 + 4x +1 – x2 + 2x – 1 – 3x2 – 6x = 0
0=0
9.
=
=
110 = 5z
Z = 22
10. =3

=3
3x2 – x + 1 = 3x2 + 3x
3x2 – x + 1 – 3x2 – 3x = 0
-4x +1 = 0
-4x = -1
X=
11. 2(x – p) = 3(6p – x) : x
2x – 2p = 18p – 3x
2x + 3x = 18p + 2p
5x = 20p
X = 4p
12. 2by – 2a = ay – 4b : y
2by – ay = 2a – 4b

Pengantar aljabar 30
y(2b – a) = 2a – 4b
y=
13. = :x
2xa – a2 = 2xb – b2
2xa – 2xb = a2 – b2
2x(a – b) = (a+b)(a-b)
X =

X =
14. = :x
X2 – xa – xd +ad = x2 – xb – xc + bc
X2 – xa – xd – x2 + xb + xc +ad = bc – ad
X(b+c – a – d) = bc – ad
X =
15. + = :y

=
Y(bc+ac) = y2ab
Y=
16. Misal : 2x = umur doni
6 tahun lalu umur doni = 6 (6 tahun lalu umur dani)
2x – 6 = 6(x – 6)
2x – 6 = 6x – 36
4x = 30
2x = 15
17. Misal : x = umur nabil
2x = umur jodi

Pengantar aljabar 31
10th yang lalu umur jodi = 4 (umur nabil 10 tahun yang
lalu)
2x – 10 = 4 (x – 10)
2x – 10 = 4x – 40
2x = 30 jodi
X = 15 nabil
Jadi umur jodi dan nabil sekarang adalah 30th dan 15th
18. (i) 2x – 3y = 7 x (1) 2x – 3y = 7
(ii)3x + y = 5 x (3) 9x + 3y = 15 +
11x = 22
X =2

Substitusikan x = 2 ke dalam (ii) 3x + y = 5


3(2) + y = 5
6+y=5
Y = -1
Jadi penyelesaian simultan dari (I) dan (ii) adalah x = 2 ,
y = -1

19. (i) 3x – y = -6 x (3) 9x – 3y = -18


(ii)2x + 3y =7 x (1) 2x + 3y = 7 +
11x = -11
X = -1
Substitusikan x = -1 kedalam (i)
3(-1) – y = -6
-3 – y = -6 Y = 3
Jadi penyelesaian simultan dari (i dan ii) adalah x = -1
y=3

Pengantar aljabar 32
20. (i) 4x + 2y = 5 x(3) 12x + 6y = 15
(ii)5x – 3y = -2 x(2) 10x – 6y = -4 +
22x = 11
X=
Substitusikan x = kedalam (i)
4( + 2y = 5
2 + 2y = 5
2y = 3
Y=
Jadi penyelesaian simultan dari (i dan ii) adalah x =
y=
21. (i) + =6 x 15 5x + 3y = 90 x 2 10x + 6y = 180
(ii) = -4 x 12 2x – 6y = -48 x 1 2x – 6y = -48 +
12x = 132
X = 11

Substitusikan x = 11 kedalam (ii)


2(11) – 6y = -48
22 – 6y = -48
-6y = -70
y = 11
Jadi penyelesaian simultan dari (i) dan (ii) adalah x = 11 dan
y =11

Pengantar aljabar 33
22. (i) + =4 x 12 8x – 4 + 3y + 6 = 48
(ii) =3 x6 3x + 9 – 2x + 8 = 18
8x + 3y = 46 x 2 16x + 6y = 92
x + 2y = 9 x 3 3x + 6y = 27 -
13x = 65
X=5
Substitusikan x = 5 kedalam (ii)
(5) + 2y = 9
2y = 4
Y =2
Jadi penyelesaian simultan dari (i) dan (ii) adalah x = 5
dan y = 2

23. 2x – 3y + z = 6 ............(1)
X + 2y + 2z = -6............(2)
4x – 5y + 3z = 10.........(3)
2x – 3y + z = 6 x2
4x – 6y + 2z = 12
X + 2y + 2z = -6 x1
X + 2y + 2z = -6 _
3x– 8y= 18...........(4)
2x – 3y + z = 6 x3 6x – 9y + 3z = 18
4x – 5y + 3z = 10 x1 4x – 5y + 3z = 10 _
2x – 4y= 8............(5)
3x – 8y = 18 x1 3x – 8y = 18
2x – 4y = 8 x2 4x – 8y = 16 _
-x =2 atau x = -2
Substitusikan x = -2 kedalam 3x – 8y =18
= 3(-2) – 8y = 18
= -6 – 8y = 18
= -8y = 24 y= -3

Pengantar aljabar 34
Substitusikan x= -2 dan y = -3 ke dalam 2x – 3y + z = 6
2x – 3y + z = 6
= 2(-2) – 3(-3) + z = 6
= -4 + 9 + z = 6
= z =1
Jadi penyelesaiannya adalah x = -2 y = -3 dan z = 1
24. X + 3y + 2z = 11....... (1)
2x +3y + z = 13.........(2)
4x + 2y +z = 17 .........(3)
Eliminasi z, dengan menggunakan persamaan (1)
dan (2) kemudian persamaan (2) dan (3)
X + 3y + 2z = 11 x1 X + 3y + 2z = 11
2x +3y + z = 13 x2 4x + 6y + 2z = 26 _
-3x – 3y = -15
-x – y = -5 ......(4)
2x +3y + z = 13
4x + 2y +z = 17 _
-2x + y = -4 ...........(5)

-x – y = -5
-2x + y = -4 +
-3x = -9
X =3
Substitusikan x = 3 ke dalam
–x – y = -5
-(3) – y = -5
- y = -2
Y=2
Substitusikan x = 3 dan y = 2 ke dalam
2x +3y + z = 11
Pengantar aljabar 35
2(3) + 3(2) + z = 13
12 + z = 13
Z =1
Jadi penyelesaiannya adalah x =3 y = 2 dan z = 1

25. 4x +2y – 3z = 1..........(1)


X – y + 3z = 5.............(2)
X + 5y – 12z = 6.........(3)
Eliminasi z dengan menggunakan persamaan (1) dan (2)
kemudian persamaan (1) dan (3)
4x +2y – 3z = 1
X – y + 3z = 5 +
5x + y = 6 ...........(4)
4x +2y – 3z = 1 x4 16x + 8y – 12z = 4
X + 5y – 12z =6 x1 X + 5y – 12z = 6 _
15x + 3 y = 2.........(5)
(4) 5x + y = 6 x3 15x + 3y = 18
(5) 15x + 3y = -2 x1 15x + 3y = -2 _
0 = 20
Sistem persamaan tidak mempunyai penyelesaian atau
disebut juga dengan sistem persamaan tidak konsisten
26. (i) X – y + 2z = 0
(ii) x +3z – 2y = -1
(iii) 2y – 2x + z = 3
Eliminasi (i) dan (ii) (i) X – y + 2z = 0
(ii) x +3z – 2y = -1 _
Y – z = 1........................(iv)
(ii) x +3z – 2y = -1
(iii) 2y – 2x + z = 3 _
3x + 2z = -4...................(v)
Pengantar aljabar 36
(i) X – y + 2z = 0
(v) 3x +2z = -4 _
-2x + y = 4......................(vi)
(i) X – y + 2z = 0
(vi) -2x + y =4 +
-x +2z = 4......................(vii)
(v) 3x + 2z = -4
(vii) -x +2z =4 _
4x = -8
X = -2
(vii) -x +2z = 4
-2 + 2z = 4
2z = 6
Z =3
Y–z=1
Y–3=3
Y =6
Jadi penyelesaian simultan dari (i) dan (ii) adalah x = 2 , y
= 6 dan z = 3

27. 3x + 4y = 13 (1)
2x +5y = 11 (2)

3x + 4y = 13 4y = -3x + 13
y= +
substitusikan y = + ke dalam (2)
2x +5y = 11
= 2x +5( + ) = 11

Pengantar aljabar 37
= 2x + = 11
= = x=3

y= +
y= +
y=1 (3,1)

28. X2 – 16 = 0
X2 = 16
X = 4
29. 4t – 9 = 0
2

4t2 = 9
t2 =
t =

30. misal x + y 120


4x + 6y 540
4x + 6y = 540 x1 4x + 6y = 540
X + y = 120 x4 4x + 4y = 480 _
2y = 60
Y = 30
Substitusikan y = 30 ke dalam x + y = 120
X + 30 = 120
X = 90 (90,30)
90000x + 100000y
90,30 90000 (90) + 100000 (30)
= 8100000 + 3000000 = 11.100000

Pengantar aljabar 38
31. Rumah tipe 1= 90 unit dan rumah tipe II = 30 unit

32. 2x + y = 6
2x + y = 6 x2 4x + 2y = 12
x + 2y = 6 x1 x + 2y = 6 _
3x =6
X=2
2x + y =6
2(2) + y = 6
Y = 2 (2,2)
f(x,y) = 5x + 3y adalah
(2,2) 5(2) + 3(2) = 16

33. 4x + y = 20
X + y = 20 _
3x = 0
X =0
X + y = 20
0 + y = 20
Y = 20 (0,20)
f(x,y) = 3x + 6y
(0,20) = 0 + 120 = 120

34. misal y = jam


50y = 45 (y + 2)
50 y = 45y + 90
5y = 90
y = 18

Pengantar aljabar 39
35. 4x + 2y = 60 x1 4x + 2y = 60
2x + 4y = 48 x2 4x + 8y = 96 _
-6y = -36
Y=6
2x + 4y = 48
2x + 4(6) = 48
2x + 24 = 48
2x = 24
X = 12

f(x,y) = 8x + 6y
(12,6) = 8(12) + 6(6)
= 96 + 36 = 132

36. misal x = modal yang akan ditambahkan dengan bunga


6 persen
4000 + x = seluruh modal pada bunga 5 persen
0,03 (3000) + 0,04 (1000) + 0,06 x = 0,05 (4000 + x)
90 + 40 + 0,06x = 200 + 0,05x
X = 7000
Jadi 7000 dengan harga 6 persen
37. misal T = jam
20 T = 30 (12 – T)
20 T = 360 – 30T
50 T = 360
T = 7,2 jam
38. 2x + 2y = 4 x2 4x + 4y = 8
6x + 4y = 36 x1 6x + 4y = 36 _
-2x = -28 X = 14
Pengantar aljabar 40
Substitusikan x = 14 kedalam 2x +2y = 4
2x +2y = 4
=2(14) + 2y = 4
= 28 + 2y = 4
= 2y = -24
= y = -12
Nilai maksimum dari p = 30x +10y
= 30 (14) + 10(-12)
= 420 – 120 = 300
39. X + 2y = 12 x2 2x + 4y = 24
2x + y = 12 x1 2x + y = 12 _
3y = 12 atau y = 4
Substitusikan x kedalam 2x + y = 12
x + 2y = 12
= x + 2(4) = 12
= x = 12
Nilai maksimum p dan q
P=x+y
= 12 + 4 = 16
Q = 5x + y
= 5(12) + 4
= 60 + 4 = 64

40. misal sapi = x ekor


Kerbau = x ekor
X + y 17 ........(1)
9.000.000 x + 8.000.000 y 144.000.000
9x + 8y 144 ........(2)
X 0 ..........(3)
y 0 .........(4)
Pengantar aljabar 41
f(x,y) = 1.3000.000 x + 1.200.000 y

9x + 8y = 144 x1 9x + 8y = 144
X + y = 17 x8 8x + 8y = 136 _
X =8
X + y = 17
8 + y = 17
Y=9 (8,9)
F(x,y) 1.300.000 x + 1.200.000 y
(8,9) 1.300.000 (8) + 1.200.000 (9)
10.400.000 + 10.800.000 = 21.200.000

41. Usia ayah = x 90


= x 90 = 48 tahun

42. misal vit B1 = x


Vit B2 = y
X + 6y 10..........(1)
3x + 3y 15
X + y 5 .....(2)
F(x,y) = 1000x +500y

X + 6y = 10 X+y=5
X+ y =5 _ X+1=5
5y = 5 x =4
Y =1
F(x,y) = 1000x +500y
(4,1) = 1000 (4) + 500 (1)
= 4000 + 500 = 4500

Pengantar aljabar 42
43. misal 5x + 3x = 30000 x3 15x + 9y = 90000
3x + 4y = 25000 x5 15x + 20y = 125000
-11y = -35000
Y=

44. Misal : Banyaknya tablet Jenis I yang diperlukan tiap hari


: x tablet
Banyaknya tablet Jenis II yang diperlukan tiap hari : y
tablet
Satu Tablet Satu Tablet Keperluan
Jenis I (x) Jenis II (y) tiap hari
Kandungan Vit. 5 10 25
A 3 1 5
Kandungan Vit.
B
Harga 4.000 8.000

Dari tabel diatas didapatkan model matematika :


x 0y
3x + y
X 0
Y 0
Dengan meminimumkan :
F (x,y) = 4.000x + 8.000y
Tititk perpotongan dua garis diselesaikan dengan
tekhnik Eliminasi dan Substitusi
Eliminasi
5x + 10y = 25 x3 15x + 30y = 75
3x + y = 5 x5 15x + 5y = 25

Pengantar aljabar 43
25y = 50
y =2
Substitusi
5x + 10y = 25
5x + 10.2 = 25
5x = 25 – 20
5x = 5 x = 1 (1,2)
F (x,y) = 4.000x + 8.000y
(1,2) = 4000(1) + 8000(2) = 20.000
nilai minimumnya adalah Rp. 20.000,00

45. Misal : Harga 1 mangkuk bakso adalah x,


Harga 1 gelas es teh adalah y. Maka, model matematika
system persamaan linearnya:
3x + 3y = 15.000
4x + 3y = 19.000
Model ini dapat diselesaikan dengan cara eliminasi dan
substitusi.

Eliminasi y :
3x + 3y = 15.000
4x + 3y = 19.000
-x = - 4000
x = 4000
substitusikan x = 4000 ke persamaan 3x + 3y =
15.000.

46. Misal : panjang = x cm dan lebarnya = y cm.


Keliling : selisih :

Pengantar aljabar 44
2p + 2l = K p – l = 6
2x + 2y = 60
x–y .
x y 0 .
model matematikanya x + y = 30 dan x – y = 6.

47. tetapkan n = jumlah tiket seharga 3


3n = jumlah tiket seharga 5
3n + 10 = jumalh tiket seharga 7,5
Sehingga 3n + 5(3n) + 7,5(3n+10) = 885
18n + 22,5n + 75 = 885
n = 20
3n = 3(20) = 60
3n + 10 = 3(20) + 10 = 70
Jadi 20 buah tiket 3, 60 buah tiket 5 dan 70 buah
tiket 7,5

48. misalkan jumlah uang setengah dollar jumlah quarter,


dan jumlah dine yang masing-masing dinyatakan
dengan n. Nilai gabungan adalah 10.
Maka 10n + 25n + 50n = 1000
85n = 1000
n =
49. Misal Harga 1 mangkuk bakso adalah x, Harga 1 gelas
es teh adalah y. Maka, model matematika system
persamaan linearnya:

3x + 3y = 15.000

4x + 3y = 19.000
Pengantar aljabar 45
Model ini dapat diselesaikan dengan cara eliminasi dan
substitusi.
Eliminasi y :

3x + 3y = 15.000

4x + 3y = 19.000

-x = – 4000

x = 4000

substitusikan x = 4000 ke persamaan

3x + 3y = 15.000.

3(4000) + 3y = 15000
12000 + 3y = 15000
3y = 3000
y = 1000

50. misal x = jumlah orang dewasa dan


Y = jumlah anak-anak.
Maka 5x + 2,5y = 122,5 x2 10x + 5y = 245
x + y = 30 x10 10x +10y = 300 _
-5y = 55
Y = 11
Substitusikan x = 11 ke dalam x + y = 30
X + y = 30
= x + 11 = 30
= x = 19
Pengantar aljabar 46
Jadi persamaannya adalah x = 19 y = 11
Atau 19 dewasa dan 11 anak-anak

Pengantar aljabar 47
BAB 3

PERSAMAAN POLINOMIAL

3.1 DEFINISI POLINOMIAL


3.2 MENYUSUN DAN MENAMBAHKAN POLINOMIAL
3.3 MENGURANGKAN POLINOMIAL
3.4 PERKALIAN POLINOMIAL
3.5 PEMBAGIAN POLINOMIAL
3.6

Pengantar aljabar 48
1.1
3. DEFINISI POLINOMIAL
1
Adalah suku ekspresi matematika dengan yang
melibatkan jumlahan perkalian pangkat dalam satu atau
lebih variabel dengan koefisien dan sebuah monomial atau
multinomial yang setiap suku adalah integral dan rasional
dalam huruf-huruf. Sebuah polinomial dalam satu variabel
dengan koefisien konstan memiliki bentuk seperti berikut:

anxn + .... + a2x2 + a1x + a0

Pangkat tertinggi pada suatu polinomial menunjukkan orde


dari polinomial tersebut.

polynomial dalam Matlab dinyatakan sebagai vektor baris


yang memuat koefisien dari variabel dengan derajat yang
makin menurun..

misal diberikan p(x) = x3 – 2x – 5 maka dalam matlab


dinyatakan dengan >> p=[1 0 -2 -5]..

Contoh : 3x2y3 – 5x4y + 2


2x4 – 7x3 -5x + 2
Adalah polinomial-polinomial.
Contoh bukan polinomial : 3x2
4√ +3

Pengantar aljabar 49
Derajat dari sebuah polinomial adalah sama dengan
derajat suku yang mempunyai derajat yang tertinggi
dan koefisien bukan nol.
Jadi 7x5y2 – 4xz6 + 2x4y mempunyai suku dengan
derajat 5,6,4 berturut-turut maka derajat polinomial
adalah 6

1.1
3. MENYUSUN DAN MENAMBAHKAN POLINOMIAL
2

Menyusun suku-suku dari suatu polinomial


dalam susunan meningkat atau menurun
Suatu polinomial dapat disusunan dengan cara sebagai
berikut:
1. Dalam susunan menurun, dengan menyusun eksponen-
eksponen dari huruf yang sama secara menurun dalam
suku-suku yang berurutan.
2. Dalam susunan menningkat dengan menyusun
eksponen-eksponen dari huruf yang sama secara
meningkat dalam suku-suku yang berurutan.
Maka, 2x2 + 3x3 – 5x + 8 menjadi
3x3 + 2x2 – 5x + 8 dalam susunan menurun
Untuk penambahan dua buah polinomial, dapat
langsung dengan tanda + (plus). Bentuk umum dalam
Mathlab adalah d=a+b dengan a dan b adalah
polynomial yang dijumlahkan dan d merupakan
polynomial hasil. Tentunya kita harus ingat bahwa
syarat penambahan dua buah polynomial adalah
ukuran dari
Pengantar aljabar 50
kedua polynomial tersebut harus sama. Sebagai
contoh : polynomial a = x3 + 2x2 + 3x + 4
ditambahkan dengan polynomial b=
x + 4 x + 9x + 16.
3 2

Jawab : a [ ]; b [ ];
d=a+b
d = 2 6 12 20
Maka hasilnya adalah polynomial 2x3 + 6x2+ 12x + 20.
Contoh : 3a + 5b , 6b -2a
Jawab : 3a + 5b
-2a + 6b +
a + 11b

1.1
3. MENGURANGKAN POLINOMIAL
3

Untuk mengurangkan polinomial


Contoh : untuk mengurangkan suatu suku
tambahkan lawannya. (suku yang berlawanan yang
berbeda tanda ) sehingga untuk mengurangkan –3x
tambahkan +3x.
Contoh : 5x – 2y dari 8x – 4y
Jawab : 8x – 4y – (5x – 2y)
= 8x – 4y – 5x + 2y
= 3x – 2y

Pengantar aljabar 51
1.1
3. PERKALIAN POLINOMIAL
4

Perkalian dua buah polynomial dalam Mathlab


didukung oleh function conv (convolution). Bentuk
umumnya adalah c=conv(a,b). Ini berarti polynomial a
dikalikan dengan polynomial b. Sebagai
Contoh :
1. polonomial a = x3 + 2x2 + 3x + 4 dikalikan dengan
polynomial
b= x3 + 4 x2 + 9x + 16 .
jawab : Dalam Mathlab diketik sebagai : a [ ];
b [ ]; c=conv(a,b)
c = 1 6 20 50 75 84 64
Jadi, c = x6 + 6x5 + 20x4+ 50x3 + 75x2 + 84x + 64.
2. 3x + 4 dikali dengan 2x -1

Jawab : 3x + 4

2x – 1 x

6x2 + 8x

-3x - 4 +

6x2 + 5x – 4

Pengantar aljabar 52
1.1
3. PEMBAGIAN POLINOMIAL
5
Kalau perkalian dua buah polynomial dengan
fungsi conv, maka dalam pembagian dengan fungsi
deconv. Bentuk umum dalam Mathlab adalah
[q,r]=deconv(c,b) dengan c sebagai polynomial
pembilang, b sebagai polynomial penyebut, q sebagai
polynomial hasil, r sebagai sisa hasil bagi.
Contoh :
1. a [ ]; b [ ];c [ 0 0 ]
jawab : Jika polynomial c dibagi b, maka :
[q,r] = deconv (c,b)
q=1234 r =0 0 0 0 0 0 0
Ini berarti hasil pembagian polynomial c=[1 6 20 50 75
] oleh polynomial b [ ]; adalah
polynomial q = [1 2 3 4] atau
q= x3 + 2x2 +3x +4 dengan sisa hasil bagi r = 0. Juga,
misalnya polynomial c dibagi oleh a,
maka : [k,l]=deconv(c,a)
k = 1 4 9 16 l=0000000
Ini berarti hasil pembagian c=[1 6 20 50 75 84 64] oleh
polynomial a [ ]; adalah polynomial q [
]; atau
q= x3 + 4x2 +9x + 16 dengan sisa hasil bagi r
= 0.

Pengantar aljabar 53
1. X2 – 5x + 6 dibagi oleh x -2
X–3
Jawab : √
x2 – 2x _
- 3x + 6
- 3x + 6 _
0
X – 5x + 6 dibagi oleh x -2 hasilnya x – 3
2

Pengantar aljabar 54
LATIHAN SOAL

1. 13a + (-2a) + (- a)
2. -2x2 + (-8x2) + (-15x2)
3. a3 + 3a3 + (-7a3)
4. xy2 + xy2 + (-xy2)
5. 2,3ab + 7,1ab + (-3,4ab)
6.
7. (-6y + 2y2 + y3 + 3y2) + (y3 + 2y2 + 3y2 -6y)
8. x3 – 3x + 3x3 – 5x2 + 4 – 8x2 – 5x + 10
9. 3x2 + 4y – 5x – 2y
10. (3b + 5) – ( 6b + 12)
11. (y2 – 3y) – (-2y2 – 4y)
12. (8x3 – 2x2 + 5x) – (6x3 + 9x)
13. 5x2 + 8x – 3x2 – 15x
14. 5x3 + 7x2 – 4 – 6x3 – 10
15. 10x + 3y + 7x – 6y + 7
16. 8a + 5b – 2a + 6b + 10b – 2
17. 2x2 + 3x4 – x3 – (-3x3 + 2x4 + x2)
18. x6y9 : x4y2
19. 15xy2 : 5xy2
20. 8a : 2a
21. 10a3 : 5a2
22. 2a3b x (-8ab)
23. 15a4b6 : 3a2b2
24. 6a2 : 4a
25. 8a3b : (-5a2)
26. 15b : 3b

Pengantar aljabar 55
27. 2a x 2a
28. 2a (2a + 8a2b – 4ab2)
29. -3b(6a2 + 5ab – 4b2)
30. -2pq(3p2 – 4pq – 7q2)
31. a2(6a2 – 2a + 5b2)
32. 16x6y8 : (4x3y2 . 2x2y4)
33. 32x5y8 : (2x3y2 . 8xy4z2)
34. 6a(3a2 – 7b)
35. 3x (2x2 + 4xy – 7y2)
36. 8p5 : (12p4 : 3p2)
37. P5q6 : (-p4q5 : p2q4)
38. (y² - 4y + 16) (y + 4)
39. (x³ + x²y + xy² + y³) (x – y)
40. (x² + 4x + 8) (x² - 4x + 8)
41. (3r – s - t²) ( 2s + r + 3t²)
42.

43.
44. ( x + x²y² + y ) ( y - x²y² + x ) dimana x = 1, y = 2
45. 3(x + 3)
46. (-a3b3)-2/3
47. -3 (-1)-1/5 (4)-1/2
48.
49. ( x+y ) (x2 + 2xy + y2)
50. 5x(x +2)

Pengantar aljabar 56
PEMBAHASAN

1. 13a – 2a – a = 10a
2. - 2x2 – 8x2 – 15x2 = - 25x2
3. a3 + 3a3 – 7a3 = 3a3
4. xy2 + xy2 – xy2 = xy2
5. 2,3ab + 7,1ab – 3,4ab = 6ab
6. = = =
7. (-6y + 2y2 + y3 + 3y2) + (y3 + 2y2 + 3y2 -6y) =2y3 +
10y2
8. 4x3 – 13x2 – 8x + 14
9. 3x2 + 2y – 5x
10. 3b +5 – 6b + 12 = - 3b + 17
11. y2 – 3y + 2y2 + 4y = 3y2 + y
12. 8x3 – 2x2 + 5x - 6x3 - 9x = 2x3 - 2x2 – 4x
13. 2x2 – 5x
14. - x3 + 7x2 – 14
15. 17x – 3y + 7
16. 6a + 21b – 2
17. x2 + 3x4 – x3 + 3x3 – 2x4 – x2 = x4 +2x2
18. = x2y7

19. =3

20. =4

21. = 2a

22. =

23. = 5a2b4

Pengantar aljabar 57
24. =

25. =
26. =5
27. 2a x 2a = 4a2
28. 2a (2a + 8a2b – 4ab2) = 4a2 + 16a3b – 8a2b2
29. -3b(6a2 + 5ab – 4b2) = -18a2b – 15ab2 + 12b3
30. -2pq(3p2 – 4pq – 7q2) = -6p3q + 8p2q2 + 14pq3
31. a2(6a2 – 2a + 5b2) = 6a4 – 2a3 + 5a2b2
32. = 2xy2

33. 32x5y8 : (16x4y6z2) =


34. 6a(3a2 – 7b) = 18a3 - 42ab
35. 3x (2x2 + 4xy – 7y2) = 6x3 + 12x2y – 21xy2
36. 8p5 : (4p2) = 2p3
37. P5q6 : (-p2q) = -p3q5
38. (y² - 4y + 16) (y + 4)
= y³ - 4y² + 16y + 4y² - 16y + 64 = y³+64
39. (x³ + x²y + xy² + y³) (x – y)
= x + x³y + x²y² + xy³ - x³y - x²y² - xy³ - y = x - y

40. (x² + 4x + 8) (x² - 4x + 8)


= x - 4x³ + 8x² + 4x³ - 16x³ - 16x² + 32x + 8x² - 32y +
64 = x + 64
41. (3r – s - t²) ( 2s + r + 3t²)
= 6rs + 3r² + 9rt² - 2s² - r5 – 3st² - 2st² - rt² - 3t
=3r² + 5rs + 8rt² - 2s² - 5st² - 3t
42. = + - 6a²c

Pengantar aljabar 58
43. 4x2 – 9x

√ –
-
-9 + 3x - 2
-9 + 9x _
12x – 2
12x + 12 _
-14

= 4x2 - 9x

44. ( x + x²y² + y ) ( y - x²y² + x )

= x x + x² y² - x² y² + y y = x⁸ + x y + y⁸

= 1⁸ + 1 2 + 2⁸ = 273

45. 3x + 9

46. √ =√ =√ =

47. - 3 . √ .√ = -3 . √ . = -3 . -1 . =
.
48. =
.

49. ( x+y ) (x2 + 2xy + y2) = x3 + 3x2y + 3xy2 + y3

50. 5x(x +2) = 10x2 + 10

Pengantar aljabar 59
BAB 4

PEMFAKTORAN

4.1 PEMFAKTORAN MONOMIAL


4.2 PEMFAKTORAN POLINOMIAL BENTUK ax2 + bx + c
4.3 PEMFAKTORAN POLINOMIAL BENTUK ax2 + bx + c
4.4 PEMFAKTORAN KHUSUS

Pengantar aljabar 60
4. PEMFAKTORAN MONOMIAL
1
Sebuah pernyataan aljabar adalah sebuah gabungan
bilangan biasa dan huruf-huruf yang dipasangkan dengan
bilangan-bilangan tersebut.

Monomial adalah suatu ekspresi matematika dengan satu


suku.
Bentuk - bentuk aljabar seperti 4a dan -5a2b disebut bentuk
aljabar suku satu atau suku tunggal atau biasa disebut
dengan monomial.

Monomial hanya mempunyai satu suku dan pemfaktoran


mononial bisa seperti.

xy2 = y(xy)

4. PEMFAKTORAN POLINOMIAL BENTUK ax2 + bx + c


2
Mefaktorkan polinomial berarti menulis polinomial menjadi
bentuk perkalian antara dua polinomial atau lebih. Polynomial
adalah suatu ekspresi matematika dengan dua atau lebih
suku.

misalkan bentuk kuadrat tersebut dapat difaktorkan


kedalam bentuk
x2 + bx + c = (x + p) (x + q)

Pengantar aljabar 61
= x2 + qx + px + pq
= x2 + (q + p)x + pq
Sehingga x2 + bx + c = x2 + (q + p)x + pq
Dimana (p + q) = b
pq = c
Contoh
1. X2 + 7x +12
Jawab
(p + q) = 7 p = 3 , q = 4
pq = 12

X2 + 7x +12 = (x + 3) (x + 4)

4. PEMFAKTORAN POLINOMIAL BENTUK ax2 + bx + c


3

misalkan bentuk kuadrat tersebut dapat difaktorkan


kedalam bentuk
ax2 + bx + c = (ax + p) (ax + q)
a2x2 + abx + ac = (ax + p) (ax + q)
= a2x2 + aqx + apx + pq
= a2 x2 + (q + p)ax + pq
Sehingga a2x2 + abx + ac = a2 x2 + (q + p)ax + pq
Dimana (p + q) = b
Pq = c

Pengantar aljabar 62
Contoh
1. 3x2 + 10x +8
Jawab
Dik : a = 3 , b = 10 , c = 8
Dit : faktornya
Jawab : terlebih dahulu dicari bilangan yang jumlahnya
10 dan hasil perkaliannya 24 yaitu 4 dan 6
Diperoleh
3x2 + 10x +8
= 3x2 + 4x + 6x + 8
= X(3x + 4) + 2(3x + 4)
= (X + 2) (3x + 4)
Jadi 3x2 + 10x +8 = (X + 2) (3x + 4)

4. PEMFAKTORAN KHUSUS
4

Beberapa di bawah ini adalah rumus-rumus khusus


yang perlu diketahui. Dan beberapa sifat pada
pemfaktoran. antara lain :
(a + b)2 = a2 + 2ab + b2
(a – b)2 = a2 – 2ab + b3
(a + b)3 = a3 + 3a2b + 3b2a + b3
(a – b)3 = a3 + 3a2b + 3b2a - b3
(a + b)4 = a4 + 4a3b + 6a2b2 + 4b3a + b4
(a - b)4 = a4 - 4a3b + 6a2b2 - 4b3a + b4
a2 - b2 = (a – b)(a + b)
a3 - b3 = (a – b)(a2 + ab +b2)
a3 + b3 = (a + b)(a2 - ab +b2)
Pengantar aljabar 63
sifat-sifat tersebut tidak perlu di hafal.

Karena dengan memperhatikan konsepnya saja, kita pasti


bisa mengingatnya dan menggunakannya pada pangkat
yang lebih besar. yang cukup penting untuk diketahui
adalah :

a2 - b2 = (a – b)(a + b)

Contoh

1. x2 – 9 = (x + 3)(x – 3)
2. x3 + 8 = x3 + 23 = (x + 2)(x2 – 2x + 4)

Pengantar aljabar 64
LATIHAN SOAL

1. 27a2b
2. 21a4bc
3. 8ab
4. 18abr2
5. 14ab3c
6. 6b2z
7. 144xy2z3
8. 169x2yz4
9. 40x3y4z
10. 100hr2
11. 2x2 – 3xy
12. 4x + 8y + 12z
13. 10a2b3c4 – 15a5b2c4 + 30a4b3c2
14. 4a n+1 – 8a 2n
15. 3x2 + 6x3 + 12x4
16. 9x3t +15x2t3 – 3x2
17. 1- n2m4
18. (2x + 3)2
19. (3x – 4)2
20. a2 + 6a + 9
21. x2 – 18x + 81
22. a2 – 6a +5
23. a2 – 16
24. x2 – 6x + 9
25. 2x2 – 6x + 4
26. x2 + 2x – 8
27. x2 – 2x – 8
Pengantar aljabar 65
28. x2 + 6x + 9
29. 25x2 – 4y2
30. X2 – 9
31. Faktorkan fungsi dari 5s – t adalah
32. Factor dari 9xy + 12y – 6xz – z adalah
33. Faktor dari 6ab – 12ac + 18ad adalah
34. Faktor dari 8ax – 20a + 10 bx – b adalah
35. Faktor dari b adalah
36. 8x3 – 27 = (2x)3 – 33 = (2x – 3)(4x2 + 6x + 9)
37. (2x + 1)³
38. (3x + 2y)³
39. (t – 2)(t² + 2t + 4)
40. (z – x)(x² + xz + z
41. (x – 2y + z)²
42. (s – 1)(s³ + s² + s + 1
43. (1 - t²)( 1 - t t - t⁶
44. (3x + 2y)² (3x – 2y)²
45. (x² + 2x + 1)² (x² - 2x + 1)²
46. (y – 1)³ (y + 1)³
47. (u + 2)(u – u u )
48. 3x + 16x + 16
2

49. 5x2 + 12x + 4


50. 3x2 + 4x – 4

Pengantar aljabar 66
PEMBAHASAN

1. 9a(3ab)
2. 7a3 (3abc)
3. 2 (4ab)
4. 6r(3abz)
5. 2b(7cab2)
6. 2b (3bz)
7. 12yz (12xyz2)
8. 13xz (13xz3)
9. 8 x2y3 (5xyz)
10. 10r(10hr)
11. 2x2 – 3xy = x (2x – 3y)
12. 4x + 8y + 12z = 4 (x + 2y + 3z)
13. 10a2b3c4 – 15a5b2c4 + 30a4b3c2 = 5a2b2c2 (2bc2 – 3a3c2
+ 6a2b)
14. 4a n+1 – 8a 2n = 4a n+1 (2an-1)
15. 3x2 + 6x3 + 12x4 = 3x2 (1 + 3x + 4x2)
16. 9x3t +15x2t3 – 3x2t2 = 3x2t (3x +5t2 – t )
17. 1 – n2m4 = 12 – (nm2)2 = (1 – nm2) (1 + nm2)
18. 4x2 + 12x + 9
19. 9x2 – 24x + 16
20. (a + 3)2
21. (x – 9)2
22. (a – 5)(a – 1)
23. (a – 4)(a + 4)
24. (x – 3)2
25. (2x –2)(x – 2)
26. (x + 4)(x – 2)
Pengantar aljabar 67
27. (x – 4)(x + 2)
28. (x + 3)2

29. 25x2 – 4y2 = (5x)2 – (2y)2 = (5x – 2y) (5x +2y)


30. X2 – 32 = (x + 3) (x – 3)
31. 5s – 5t = 5(s – t)
32. 9xy + 12y – 6xz – 8z = 3y(3x + 4) – 2z(3x + 4)
33. 6ab – 12ac + 18ad = 6a(b – 2c + 3d)
34. 8ax – 20a + 10 bx – 25b = 2a(4x – 10) + 5b(2x – 5)
35. Carilah faktor persekutuan dari 6b dan 8. Kamu telah
mengetahui bahwa FPB dari 6 dan 8 adalah 2,
kemudian bagilah setiap suku dengan FPB tersebut.

=4
Jadi, 6b + 8 = 2(3b + 4).
36. (2x)3 – 33 = (2x – 3)(4x2 + 6x + 9)
37. (2x)³ + 3(2x)² (1) + 3(2x)(1)² + (1)³ = 8x³ + 12x² +
6x + 1
38. (3x)³ + 3(3x)²(2y) + 3(3x)(2y)² + (2y)³
= 27x³ + 54x²y + 36xy² + 8y³
39. t (t² + 2t + 4) – 2(t² + 2t + 4)
= t³ + 2t² + 4t – 2t² - 4t- 8
= t³ - 8
40. z (x² + xz + z²) – x (x² + xz + z²)
= xz² + xz² + z³ - x³ - x²z - xz²
= z³ - x³
41. (x)² + (-2y)² + (z)² + 2(x)(-2y) + 2(x)(z) +2(-2y)(z)
= x² + 4y² + z² - 4xy + 2xz – 4yz
= x² - 4xy + 4y² + 2xz – 4yz + z²

Pengantar aljabar 68
42. s (s³ + s² + s + 1) – 1 (s³ + s² + s + 1)
s s s s - s³ - s² - s – 1
s -1
43. 1 (1 - t t - t⁶ t -t t - t⁶
=1-t t - t⁶ t - t t⁶ - t⁸
= 1 - t⁸
44. [(3x + 2y)(3x + 2y)] [(3x – 2y)(3x – 2y)]
= (9x² + 12xy + 4y²)(9x² - 12xy + 4y²)
= (9x² + 4y² + 12xy)(9x² + 4y² - 12xy)
= (9x² + 4y²)² - (12xy)²
x x y y - 144x²y²
x - x y y
45. [(x² + 1 + 2x)(x² + 1 – 2x)]²
= [(x² + 1)² - x ] [x x – 4x²]²
x - 2x² + 1)²
x - x x - x x
2(-2x²)(1)
x⁸ x – x⁶ x - 4x²
x⁸ - x⁶ x - 4x² - 1
46. [(y – 1)(y + 1)]³ = (y² - 1²)³
= (y²)³ - 3(y²)² 1² + 3(y²) (1²)² - (1²)³
y⁶ - y y -1
47. (u² - u u
= (u² - u u
u - u u⁸ - 16²)
u⁸ - 256
48. 3x + 16x + 16 = 3x2 +12x + 4x +16
2

= (3x2 + 12x) + (4x + 16)


= 3x(x + 4) + 4(x + 4)
= (3x + 4) (x + 4)
Pengantar aljabar 69
49. 5x2 + 12x + 4 = 5x2 + 10x + 2x + 4
= (5x2 + 10x) + (2x + 4)
= 5x(x + 2) + 2(x + 2)
= (5x + 2) (x + 2)
50. 3x + 4x – 4 = 3x2 + 6x – 2x – 4
2

= (3x2 + 6x) – (2x + 4)


= 3x(x + 2) – 2(x + 2)
= (3x – 2)(x + 2)

Pengantar aljabar 70
BAB 5

KONSEP-KONSEP ALJABAR PECAHAN

5.1 Operasi Pada Bilangan Pecahan


1. Perkalian pecahan
2. Pembagian pecahan
3. Penjumlahan dan pengurangan pecahan
4. Pecahan kompleks

Pengantar aljabar 71
Pecahan rasional aljabar adalah pernyataan yang dapat
ditulis sebagai hasil bagi dua polinomial, . p disebut
pembilang dan q disebut penyebut pecahan.

2.1.2 OPERASI PADA BILANGAN PECAHAN


1

1. Perkalian pecahan

Hasil perkalian dua pecahan atau lebih menghasilkan


sebuah pecahan yang pembilangnya adalah hail kali
pembilang-pembilang dari pecahan yang deberikan dan
penyebutnya adalah hasil kali penyebut-penyebut pecahan
yang diberikan.
Contoh soal :
Sederhanakan bentuk-bentuk perkalian berikut.
1.
2.
Penyelesaian :
1.

Pengantar aljabar 72
2. pembagian pecahan

Untuk pembagian dua pecahan, berlaku membagi


dengan suatu pecahan sama artinya mengalikan dengan
kebalikan pecahan pembagi.
Contoh soal :
Sederhanakan bentuk-bentuk pembagian berikut.
1.
2.

Penyelesaian:
1.
2.

3. Penjumlahan dan pengurangan pecahan

Hasil operasi penjumlahan dan pengurangan pecahan


diperoleh dengan cara menyamakan penyebutnya terlebih
dahulu, kemudian menjumlahkan atau mengurangkan
pembilangnya.
Contoh soal:
1.

2.

Penyelesaian:

Pengantar aljabar 73
1.
2.

4. Pecahan kompleks

Pecahan kompleks adalah pecahan yang pembilang atau


penyebutnya atau kedua-duanya mempunyai satu atau lebih
pecahan. Cara menyederhanakan pecahan kompleks adalah :
 Jabarkan pembilang dan penyebut menjadi pecahan
yang sederhana.
 Bagi kedua hasil pecahan.

Contoh soal :

1.

Penyelesaian :

1.

Pengantar aljabar 74
LATIHAN SOAL :

1.

2.

3.

4.

5.

6.

7.
8.
9.
10.
11.

12.

13.

14.
15.

16.

Pengantar aljabar 75
17.
18.

19.

20.

21.

22.
23.

24.

25.

26.

27.

28.

29.

30.

31.

32.

Pengantar aljabar 76
33.

34.

35.

36.

37.

38.

39.

40.

41.

42.

43.

44.

45.

46.

47.

48.

Pengantar aljabar 77
49.

50.

Pengantar aljabar 78
PEMBAHASAN

1.
2.

3.

4. =

5. = -2ab

6.

7.

8.

9.

10.
11.

12.

13.

14.

Pengantar aljabar 79
15.

16.

17.

18.

19.

20.

21.

22.

23.

24.

25.

26.

27.

28.

Pengantar aljabar 80
29.

30.

31.

32.

33.

34.

35.

36.

37.

38.

39.

40.

Pengantar aljabar 81
41.


42.

43.

44.

45.

46.

47.

48.
49.

50.

Pengantar aljabar 82
BAB 6

GRAFIK PERSAMAAN LINEAR

6.1 SISTEM KOORDINAT KARTESIUS


6.2 GAMBAR GARIS LURUS
6.3 TITIK POTONG GARIS LURUS
6.4 PERSAMAAN GARIS LURUS

Pengantar aljabar 83
6. SISTEM KOORDINAT KARTESIUS
1

Koordinat Cartesius yang memiliki sumbu mendatar


(disebut sumbu-x) dan sumbu tegak (disebut sumbu-y).
Titik potong kedua sumbu tersebut dinamakan titik asal
atau titik pusat koordinat

a. Menggambar Titik pada Koordinat Cartesius

Setiap titik pada bidang koordinat Cartesius


dinyatakan dengan pasangan berurutan x dan y, di
mana x merupakan koordinat sumbu-x (disebut absis)
dan y merupakan koordinat sumbu-y (disebut ordinat).
Jadi, titik pada bidang koordinat Cartesius dapat
dituliskan (x, y). Pada Gambar 3.2 , terlihat ada 6 buah
titik koordinat pada bidang koordinat Cartesius.
Dengan menggunakan aturan penulisan titik koordinat,
keenam titik tersebut dapat dituliskan dalam bentuk
sebagai berikut.

Pengantar aljabar 84
b. Menggambar Garis pada Koordinat Cartesius
Kamu telah memahami bagaimana menggambar titik
pada bidang koordinat Cartesius. Sekarang bagaimana
menggambar garis lurus pada bidang yang sama? Coba
perhatikan Gambar 3.3

Pengantar aljabar 85
Perlu diingat, garis lurus adalah kumpulan titik-titik yang
letaknya sejajar. Dari Gambar 3.3(a) , terlihat bahwa titik-titik
P, Q, R, S, T, dan U memiliki letak yang sejajar dengan suatu
garis lurus, misalkan garis k, seperti yang digambarkan pada
Gambar 3.3(b). S ebuah garis lurus dapat terbentuk dengan
syarat sedikitnya ada dua titik pada bidang koordinat
Cartesius.

6. GAMBAR GARIS LURUS


2

Persamaan garis lurus adalah suatu persamaan yang


jika digambarkan ke dalam bidang koordinat Cartesius
akan membentuk sebuah garis lurus. Cara menggambar
persamaan garis lurus adalah dengan menentukan nilai x
atau y secara acak. Perlu diingat bahwa dua titik sudah
cukup untuk membuat garis lurus pada bidang koordinat
Cartesius.

6. TITIK POTONG GARIS LURUS


3

Titik potong garis lurus adalah cara menentukan


persamaan garis yang saling sejajar maupun tegak lurus.
Dua garis yang sejajar tidak akan pernah berpotongan di
satu titik. Sebaliknya, dua garis yang saling tegak lurus
pasti berpotongan di satu titik. Dengan tanpa
Pengantar aljabar 86
menggambarnya terlebih dahulu, kita dapat
menentukan titik potong dua garis yang tidak sejajar.
Pelajari uraian berikut.
Kedudukan dua garis pada bidang

a. Dua buah garis yang sejajar

b. Dua buah garis yang tegak lurus atau berpotongan

Pengantar aljabar 87
6. PERSAMAAN GARIS LURUS
4

bagaimana menggambar persamaan garis lurus pada


bidang koordinat Cartesius dan menentukan gradien dari
suatu persamaan garis. gradien yang diperoleh dari
perbandingan ordinat dan absis? Bentuk tersebut dapat
dituliskan sebagai berikut.

Bentuk y = mx merupakan bentuk persamaan garis lurus


sederhana. Dikatakan sebagai bentuk sederhana karena
garis yang dibentuk oleh persamaan garis tersebut selalu
melalui titik pusat koordinat

Adapun bentuk umum dari persamaan garis lurus dapat


dituliskan sebagai berikut.

Persamaan garis ini hampir sama dengan bentuk


sederhananya, namun diberi tambahan konstanta (diberi
lambang c). Hal ini menunjukkan bahwa garis yang
dibentuk oleh persamaan garis tersebut tidak akan
melalui titik O(0, 0). Setelah kamu memahami bentuk
sederhana dan bentuk umum persamaan garis, berikut ini

Pengantar aljabar 88
akan diuraikan bagaimana menentukan sebuah
persamaan garis dari titik koordinat atau gradie

a. Menentukan Persamaan Garis dari Gradien dan Titik


Koordinat
Sekarang, coba kamu perhatikan Gambar 3.1.
Gambar tersebut menunjukkan sebuah garis k pada
bidang koordinat Cartesius. Garis tersebut melalui titik
A(x1, y1) dan tidak melalui titik pusat koordinat
sehingga persamaan garis pada Gambar 3.11 dapat
dituliskan:
y1 = mx1 c .

Adapun bentuk umum persamaan garis yang


tidak melalui titik pusat koordinat dituliskan:
y mx c .

Jika ditentukan selisih dari persamaan (2) dan


persamaan (1) maka diperoleh:

Pengantar aljabar 89
Selanjutnya diperoleh rumus umum untuk
menentukan persamaan garis jika diketahui gradien
dan titik koordinat, yaitu:

b. Menentukan Persamaan Garis yang Melalui Dua Titik

Pada bagian sebelumnya, kamu telah


mempelajari cara menentukan persamaan garis yang
melalui satu titik koordinat dan gradiennya diketahui.
Sekarang, kamu akan mempelajari bagaimana
menentukan persamaan garis yang melalui dua titik.
Caranya hampir sama dengan rumus umum yang telah
dipelajari sebelumnya.
Coba kamu perhatikan uraian berikut :
• y – y1 = m (x – x1) adalah rumus umum persamaan

Pengantar aljabar 90
garis dari gradien dan titik koordinat.

Jadi, rumus untuk menentukan persamaan garis yang


melalui dua titik koordinat adalah

Pengantar aljabar 91
LATIHAN SOAL

1. Perhatikan titik-titik pada koordinat katesius di bawah


ini !

Tentukanlah titik

koordinat tersebut..

2. Diketahui segiempat ABCD dengan koordinat titik A(-2,


5), B(-2, 1), C(4, 1), dan D(4,5). Segiempat ABCD
berbentuk....

3. tentukantitik koordinat
yang sesuai dengan
h gambar disamping

Pengantar aljabar 92
4. Perhatikan gambar di bawah ini

Koordinat titik Q adalah..

5. Perhatikan gambar di bawah ini

Koordinat titik C dan D berturut-


turut C(4, -3) dan D(4, 1), bangun
ABCD disebut...

6. Gambarlah titik-titik dengan koordinat berikut ini pada


bidang koordinat kartesius!
a. (4 , 3) b. (-2, -2)
7. Gambarlah titik-titik dengan koordinat berikut ini pada
bidang koordinat kartesius!
a. (3, -2) b.(-2, 2) c. (-3 , -1)
8. Diketahui titik-titik pada bidang koordinat kartesius
sebagai berikut.
a.(-2 , 2) b.(1 , 4) c.(5 , 1) d.(3 , -2) e.(-3 , -1)

Pengantar aljabar 93
9. Tentukanlah titik-titik berikut pada bidang koodinat
kartesius

10. Tentukan absis dan ordinar titik-titik koordinat berikut.


a. (3 , 4) b. (-5 , 3) c. (4 , -3) d. (9 , 5) e. (-6 , 10)
11. Diketahui titik-titik pada bidang koordinat Cartesius
sebagai berikut.
a. (10, –5) b. (–7, –3) c.(–4, 9) d.(2, 8) e.(6, 1)
Tentukan absis dan ordinat dari masing-masing titik
tersebut.
12. perhatikan gambar bidang koordinat kartesius di bawah
ini. Kemudian tentukan titik koordinat dari masing-
masing titik tersebut.

Pengantar aljabar 94
13. dalam satu bidang koordinat kartesius. Gambarkan titik-
titik berikut ini :
p(5, -2) S(3, 5) Q (-3, -1) T (0, -4) R (-4, 3)

14. Tentukan apakah titik-titik berikut membentuk garis


lurus atau tidak?
a. A(0 , 0) B(1 , 1) C(2 , 2)
a. D(2, –2) E(1 , –1) F(0 , 0)
15. Tentukan apakah titik-titik berikut membentuk garis
lurus atau tidak?
a. G(–2 , 1) H(1 , 0) I(4, 3)
b. J(2 , –2) K(3 , 0) L(1, 1)
16. sebuah titik terletak pada absis 8 dan ordinat -3.
Penulisan yang benar untuk koordinat titik tersebut
adalah...
17. perhatikan gambar bidang koordinat kartesius berikut ini.

Dari gambar tersebut, titik yang memiliki ordinat yang


sama adalah titik...
Pengantar aljabar 95
18. berikut ini adalah titik koordinat yang dilalui oleh garis
, adalah...
19. berikut ini adalah titik koordinat yang dilalui oleh garis
, adalah...
20. gambarlah garis lurus dengan titik koordinat yang dilalui
oleh garis adalah...
21. Gambarkan garis lurus yang melalui titik P(3, –3) dan Q(–
3, 3) adalah...
22. Persamaan garis melalui titik (–2, 5) dan sejajar garis
x - 2y 0 adalah .
23.

Lihatlah gambar di atas, berapakah garis lurus yang di


atas, adalah...

24. gambar lah garis lurus dari titik –titik berikut :


a. (0,1) b. (1, 2) c. (2, 3) d. (3, 4)
e. (-1, 0) f. (-2, -1) g. (-3, -2) h. (-4, -3)

25. berikut ini gambarlah titik koordinat yang dilalui oleh


garis ...

Pengantar aljabar 96
26. Gambarkan garis lurus yang melalui titik P(4, –4) dan
Q(–4, 4) adalah...
27. Gambarkan garis lurus yang melalui titik P(2, –2) dan Q(–
2, 2) adalah...
28. gambar lah titik (0,0), (1, 1), (2,2)membentuk garis lurus
...
29. 27. . gambar lah titik (2, -2), (1, -1), (0,0)membentuk garis
lurus..
30. tentukan titik potong antara garis dan garis
.
31. gambarkan persaman garis lurus dari (-4, 4) dan (-2, 2)
32. tentukan persamaan garis melalui titik ( 0, 5) yang
bergradien -2
33. tentukan persamaan garis melalui titik (4 , -6) dan
bergradien 5.
34. persamaan garis yang melalui titik (5, -5) dan -5, 1)
adalah ...
35. persamaan garis yang sejajar dengan garis 2x – y +6 = 0
dan melakukan titik (4,-1) adalah....
36. tentukan persamaan garis yang tegak lurus dengan garis
x – 2y + 3 = 0 dan melalui titik (- 4, 5)
37. persamaan garis yang melalui titik ( -2, 4) dan (6, 3)
adalah...
38. persamaan garis yang melalui titik (-2, -3) dan tegak lurus
dengan garis yang persamaan 0 adalah..
39. dari persamaan – persamaan garis berikut :
(i) y = 3x – 2 (iii) y = 10 – x
(ii) y = 2x + 3 (iv) y = 4x – 5
40. gambarlah grafik dari persamaan !

Pengantar aljabar 97
41. gambar lah garis dengan persamaan !
42. gambar lah garis dengan persamaan !
43. gambar lah garis dengan persamaan !
44. gambar lah garis dengan persamaan 0!
45. Gradien garis yang melalui titik (5, -3) dan (3 , -8) adalah?
46. Grafik persamaan 3x – 2y = 12 dan 5x + y = 7 berpotong
di titik (p , q) nilai 4p + 3q =
47. Persamaan garis yang melalui titik (2,3) dan sejajar
dengan garis yang persamaannya 3x + 5y = 15
48. Persamaan garis yang melalui titik (2,5) dan tegak lurus
dengan garis x – 2y + 4 = 0
49. Persamaan garis yang melalui titik (3 ,-5) dan sejajar
dengan garis yang persamaannya 5x - 2y = 8
50. Persamaan garis yang melalui titik (4 ,-2) dan tegak lurus
dengan garis 3y = 7 – 6x

Pengantar aljabar 98
PEMBAHASAN

1. A.(-3 , 1) B.(-1 , 3) C. (3 , -1) D.(1 , -3)


2. Persegi panjang
3. A(-1 , 4) B.(4 , 5) C(6 , -2) D(1 , -4)
4. Q(3,1)
5. Persegi panjang
6.

7.

Pengantar aljabar 99
8.

9. A.(0 , 2) b.(1 , 3) c.(4 , 1)

10. a. Dari titik (3 , 4) diperoleh absis : 3, ordinat 4


b. Dari titik (-5 , 3) diperoleh absis : -5, ordinat 3
c. Dari titik (4 , -3) diperoleh absis : 4, ordinat -3
d. Dari titik (9 , 5) diperoleh absis : 9, ordinat 5
e. Dari titik (-6 , 10) diperoleh absis : -6, ordinat 10

11. a. Dari titik (10, –5) diperoleh absis: 10, ordinat: –5


b. Dari titik (-7 , -3) diperoleh absis: -7, ordinat: -3
c. Dari titik (–4 , 9) diperoleh absis:–4, ordinat: 9
d. Dari titik (2 , 8) diperoleh absis: 2, ordinat: 8
e. Dari titik (6 , 1) diperoleh absis: 6, ordinat: 1

12. a. (0 , 0) b. (1 , 2) c.(5 , 1) d. (3 , 3) e. (-3 ,-2)


f. (-1 , 3) g. (-4 , 1) h. (-1 , -2) i. (4 , -2) j. (2 , -1)

Pengantar aljabar 100


13.

14.

15.

Pengantar aljabar 101


16. (8 , -3)
17. A dan C
18. (3, 6)
19. (0,4) dan (3, 1)
20. misal x = 0 y = -2
x = 1 y = -1
x=2y=0

21.

22. x - 2y + 2 = 0, maka a = 1 dan b = -2


m1 = -a/b = -1/-2 = 1/2
karena kedua garis sejajar maka m2 = m1 = 1/2
melalui (-2,5) maka x1 = -2 dan y1 = 5

Pengantar aljabar 102


y - y1 = m( x - x1)
y - 5 = 1/2 (x + 2)
2y - 10 = x + 2
2y - x = 12 atau x - 2y = -12
23. (-5, 5) dan (5, -5)
24.

25. Misal x = 0 maka y = -2, jadi : (0, -2)


X = 3 maka y = 1, jadi : (3, 1)

Pengantar aljabar 103


26.

27.

28.

Pengantar aljabar 104


29.

30. a. garis .
Untuk x = 1 , y =2 titik (1,2)
Untuk x =0 , y =5 titik (0, 5).
b. garis
untuk x = 5 , y = 1 titik (5,1)
untuk x = -1 , y = -3 titik (-1,-3).
gambarlah grafik dari titik –titik tersebut

Pengantar aljabar 105


Dari gambar dapat dilihat bahwa koordinat titik potong
dua garis tersebut adalah titik A(2, -1).
31.

32. Gradien : -2 maka m = -2


Melalui ( 0, 3) maka C = 3
Persamaan garisnya adalah y = -2x +3
33. Y - y₁ m x - x₁
Y +6 = 5 (x – 4 )
Y +6 = 5x -20 y =5x -26
34. (5, -5) dan (-5, 1)
₁ ₁
Persamaan garis ₁ ₁

-10(y+5) = 6(x – 5)
-10y - 5 = 6x – 30
y =
y = atau
5y = -3x – 10 atau
3x + 5y + 10 = 0
35. Garis 2x – y + 6 = 0
-y = -2x – 6
Pengantar aljabar 106
y = 2x + 12 m₁
m m₁ 2
m = 3 melalui titik (2, -1) adalah ...
y + 1 = 2(x – 2)
y +1 = 2x – 4
y = 2x – 5
-2x + y + 5 = 0 atau
2x – y – 5 = 0
36. Garis x – 2y + 3 = 0
-2y = -x – 3
2y = x +3 m₁
m - karena tegak lurus m₁ - m -1)
m -2 melalui titik (-4, 5) adalah...
y -5 =
y–5=
y=
y= atau 2x + y +3 = 0
37. 0 0
38. 0
39. (i), (ii), (iii)
40. Persamaan
Jika x = 0 maka 0 0 (0, 0).
Jika x = 4 maka (4, 2)

Pengantar aljabar 107


41. Persamaan
Jika x = 0, Titiknya adalah (0, - 3).
Jika y = 2, x = 1 Titiknya adalah (1 , 2)

42. Persamaan
Jika x = 0, y = 2 Titiknya adalah (0, 2).
Jika x = 1, y = 5 Titiknya adalah (1, 5)

Pengantar aljabar 108


43. Persamaan
Jika x = 0, y = 0 Titiknya adalah (0, 0).
Jika x = 2, y = -4 Titiknya adalah (2, -4)

44. Persamaan 0
Jika x = 0, maka 0 , y=2 Titiknya (0,
2).
Jika x = 2, maka . 0
0
y 0
Pengantar aljabar 109
Titiknya adalah (2, 3)

45. Dik : (5 , -3) dan (3 , -8)


Dit : gradien
Jwb: m =

=
= =
46. (i) 3x – 2y = 12
(ii) 5x + y = 7
Jawab : (ii) 5x + y = 7
Y = 7 – 5x
Substitusikan kedalam (i)
3x – 2y = 12
= 3x – 2(7 – 5x) = 12
= 3x – 14 + 10x = 12
= 13x = 26
= x=2

Pengantar aljabar 110


Substitusikan x = 2 kedalam (ii)
5x + y = 7
= 5(2) + y = 7
= 10 + y = 7
= y = -3 (2 , -3)
(x , y) = (p , q)
4p + 3q
= 4(2) + 3(-3)
= 8 – 9 = -1
47. Persamaan garis 3x + 5y = 15 dan titik (2 , 3)
m=
y – y1 = m (x – x1)
=y–3= (x – 2)
= 5y – 15 = -3x + 6
= 3x + 5y = 21
48. Persamaan garis X – 2y + 4 = 0 titik (2 , 5)
m1 =
karena m1 m2 maka m2 = -2
y – y1 = m (x – x1)
= y – 5 = -2 (x – 2)
= y – 5 = -2x + 4
= 2x + y – 9 = 0
49. Persamaan garis 5x – 2y = 8 dan titik (3 , -5)
m=
y – y1 = m (x – x1)
= y – ( - 5) = (x – 3)
= 2y + 10 = 5x – 15
= 5x + 2y = 25 atau 5x + 2y – 25 = 0
Pengantar aljabar 111
50. Persamaan garis 6x + 3y – 7 = 0 titik (4 , -2)
m1 = -2
karena m1 m2 maka m2 =
y – y1 = m (x – x1)
= y – (-2) = (x – 4)
= 2y + 4 = x - 4
= 2y – x + 8 = 0

Pengantar aljabar 112


BAB 7

SISTEM PERSAMAAN LINIER

7.1 DEFINISI PERSAMAAN LINEAR


7.2 METODE PENYELESAIAN SISTEM PERSAMAAN
1. Metode substitusi
2. Metode eliminasi
3. Metode grafik
7.3 MODEL MATEMATIKA SISTEM PERSAMAAN LINEAR

Pengantar aljabar 113


1.1
7. DEFINISI PERSAMAAN LINEAR
1

Sistem persamaan linear adalah persamaan yang


memunculkan variabel-variabel bentuk tunggal berpangkat
satu dan sistem yang melibatkan dua buah persamaan linear
atau yang melibatkan dua buah persamaan linear atau lebih
yang saling berkaitan

Ax +by +cz = d

1.1
7. METODE PENYELESAIAN SISTEM PERSAMAAN
2
i. metode substitusi
ii. metode eliminasi
iii. metode grafik

1. Metode substitusi

Dalam metode substitusi , salah satu variabelnya


dipisahkandari satu persamaan yang ada (satu persamaan
dinyatakan ke dalam bentuk eksplisit seperti y = mx + k
atau x = my + k ), kemudian variabel yang dipisahkan ini
disubstitusikan ke persamaan yang satunya lagi.
Contoh:
1. Diketahui sistem persamaan : 3x + y = 5
2x + 3y = 8
Tentukan penyelesaiannya denagn metode substitusi
Pengantar aljabar 114
3x + y = 5 y = -3x + 5
2x + 3y = 8 2x + 3 (-3x + 5) = 8
2x – 9x + 15 = 8
-7x = -7
x=1

y = - 3x + 5 = -3(1) + 5 = 2
jadi penyelesaian sistem persamaan adalah x = 1 dan
y=2

2. Metode eliminasi

Dalam metode eliminasi , salah satu variabelnya


dieliminasi atau dihilangkan dengan cara mengurangkan
kedua persamaan yang ada. Agar variabel yang
diinginkan tereliminasi pada saat dikurangkan maka
sebelumnya koefisien variabel tersebut harus bernilai
sama atau disamakan.

Contoh
Diketahui sistem persamaan : 2x + 3y = 11
5x + 3y = 23
Tentukan penyelesaianny adengan metode eliminasi
Jawab : 2x + 3y = 11
5x + 3y = 23 -
-3x = -12
x=4

Pengantar aljabar 115


2x + 3y = 11 x5 10x + 15y = 55

5x + 3y = 23 x2 10x + 6y = 46 -

9y = 9

Y =1

Jadi penyelesaian sistem persamaan adalah x = 4 dan y = 1

3. metode grafik

secara geometri sistem persamaan tersebut merupakan


persamaan dua buah garis lurus, letak dua garis lurus
pada bidang ada tiga kemungkinan, yaitu
1. berpotongan
2. sejajar
3. berimpit

untuk lebih jelasnya perhatikan gambar dibawah ini

P(x1y1) adalah titik potongan garis l dan k

Pengantar aljabar 116


Dari gambar diatas didapat
Dua buah garis berpotongan , jika terdapat satu titik yang
terletak pada kedua garis itu dan disebut titik
persekutuan. Atau koordinat titik potong kedua garis
merupakan himpunan penyelesaian dari sistem
persamaan linear dua variabel
Jadi, untuk garis l dan k yang mewakili sistem persamaan
ax + by = c
px+ qy = r berlaku
1. jika ≠ maka l berpotongan dengan k

2. jika ≠ maka l sejajar dengan k

3. jika maka l berimpit dengan k

contoh

carilah titik potong antara garis 2x + y = 6 dan x + y = 4


kemudian tunjukkan pula grafiknya

 2x + y = 6

Untuk x = 0 y = 6 (0,6)

Y = 0 x = 3(3,0)

2x + y = 6 (3,6)

 X+y = 4

Pengantar aljabar 117


Untuk x = 0 y = 4 (0,4)
Y = 0 x= 4 (4,0)
X+y = 4 (4,4)
2x + y = 6 x+y=4
X +y=4 - x+y=4
X=2 y =2

Jadi titik potongnya (2,2)

1.1
7. MODEL MATEMATIKA SISTEM PERSAMAAN LINEAR
3
Dalam kehidupan sehari-hari , perhitungan secara
matematika sangat diperlukan. Kadang ada suatu masalah
yang dapat diterjemahkan ke dalam model matematika
yang berbentuk sistem persamaan. Sebelum dapat
menentukan masalah ke dalam model matematika perlu
adanya pengidentifikasian atau perumusan masalah
berikut ini akan dibahas mengenai model matematika
berbentuk sistem persamaan.

Contoh

Perbandingan usia tigor dan usia anton adalah 3:4 pada


saatini, enam tahun kedepan, perbandingan usia tigor dan
anton menjadi 5 : 7 berapakah usi tigor dan anton saat
ini?
Karena itu x , y = 3 : 4 atau =

Pengantar aljabar 118


=
4x = 3y 4x – 3y = 0 ..... (1)
Enam tahhun kemudian usia tigor adalah x + 6 dan usia
anton adalah y + 6 , dam perbandingan usia mereka
adalah 5 : 7
Karena itu (x + 6) : (y + 6) = 5 : 7 atau
=
7x + 42 = 5y + 30
7x – 5y = - 12 .... (2)
Usia tigor dan anton harus memenuhi hubungan 1 dan 2
sehingga diperoleh
4x + 3y = 0
7x – 5y = -12
Sehinggga membentuk suatu sistem persamaan yang yang
disebut sistem persamaan linear
Dalam pokok bahasan ini akan diuraikan mengenai sistem
persamaan linear dengan dua variabel

Pengantar aljabar 119


LATIHAN SOAL

1. Carilah himpunan penyelesaian (HP) dari sistem


persamaan berikut ini
2x + y = 8
X+y=6
2. Carilah himpunan penyelesaian sistem persamaan
x+ 3y = 8
x+y=2
3. Carilah himpunan penyelesaian sisitem persamaan
5x + 4y = 24
2x – 3y = 5
4. Carilah himpunan penyelesaian dari sistem persamaan
berikut dengan cara substitusi
X+y=7
2x – y = 2
5. Carilah himpunan penyelesaian dari sistem persamaan
berikut dengan cara substitusi
2x – 5y = 1
4x – 3y = 9
6. Carilah himpunan penyelesaian dari sistem persamaan
berikut dengan cara eliminasi
X+y=7
2x – y = 2
7. Carilah himpunan penyelesaian dari sistem persamaan
berikut dengan cara eliminasi
2x – 5y = 1
4x – 3y = 9
8. Carilah himpunan penyelesaian dari sistem persamaan
berikut
Pengantar aljabar 120
4x + 3y = 12
8x – 5y = 2
9. Carilah himpunan penyelesaian dari sistem persamaan
berikut 2x + 3y = 16 dan 3x – y = 2
10. Carilah himpunan penyelesaian dari sistem persamaan
berikut 0 dan 0
Tentukanlah nilai nilai
11. Carilah himpunan penyelesaian dari sistem persamaan
berikut dan
12. Carilah himpunan penyelesaian dari sistem persamaan
berikut dan 0
13. Tentukan penyelesaian dari x + y = 4 dan x – 2y = -2
dengan metode substitusi!
14. Tentukan penyelesaian dari x + y = 4 dan x – 2y = -2
dengan metode eliminasi!
15. Harga 1 kg beras dan 4 kg minyak goreng Rp
14.000,00. Sedangkan harga 2 kg beras dan 1 kg
minyak goreng Rp10.500,00. Tentukan
Modelmatematika dari soal tersebut
16. Harga 1 kg gula dan 6 kg kacang ijo Rp 16.000,00.
Sedangkan harga 2 kg gula dan 1 kg kacang ijo
Rp10.000,00. Tentukan Harga sebuah gula dan kacang
ijo
17. Carilah himpunan penyelesaian dari sistem persamaan
berikut dan
18. Carilah himpunan penyelesaian dari sistem persamaan
berikut dan
19. Carilah himpunan penyelesaian dari sistem persamaan
berikut 0 dan

Pengantar aljabar 121


20. Carilah himpunan penyelesaian dari sistem persamaan
berikut
21. Carilah himpunan penyelesaian dari sistem persamaan
berikut dan
22. Carilah titik potong antara garis X + 3y = 8 dan x + y
=2
23. Carilah titik potong antara garis 5x + 4y = 24 dan 2x –
3y = 5
24. Carilah himpunan penyelesaian dari sistem persamaan
berikut x + y = 7 dan 2x + y = 9 -
25. Carilah himpunan penyelesaian dari sistem persamaan
berikut 2x + 5y = 10 dan x – 2y = 5
26. Carilah himpunan penyelesaian dari sistem persamaan
berikut x – y = - 1 dan 3x + 2y = - 13
27. Carilah himpunan penyelesaian dari sistem persamaan
berikut x – 3y = 8 dan 3x - y = -8
28. Carilah himpunan penyelesaian dari sistem persamaan
berikut 2x + y = 9 dan x + 2y = 3
29. Carilah himpunan penyelesaian dari sistem persamaan
berikut 3x – 2y = 6 dan 4x + 2y = 22
30. Carilah himpunan penyelesaian dari sistem persamaan
berikut x + y = 1 dan 2x + y = -2
31. Carilah himpunan penyelesaian dari sistem persamaan
berikut x + y = 5 dan x – y = -1
32. Carilah himpunan penyelesaian dari sistem persamaan
berikut x + 4y = 3 dan 2x + y = 20
33. Carilah himpunan penyelesaian dari sistem persamaan
berikut 5x + y = 7 dan 2x – 3y = 13
34. Dengan metode grafik, tentukan penyelesaian sistem
persamaan x + y = 6 dan 2x – y = 0
Pengantar aljabar 122
35. Tentukan penyelesian sistem persamaan x + y = 10
dan x – y = 2 dengan metode eliminasi.
36. Tentukan penyelesian sistem persamaan x – y = 1 dan
x + y = 7 dengan metode eliminasi.
37. Tentukan penyelesian sistem persamaan x – y = 6 dan
2x + y = 18 dengan metode eliminasi.
38. Umur sani 7 tahun lebih tua dari umur Ari. Sedangkan
jumlah umum mereka adalah 43 tahun. Tentukanlah
Model matematika dari soal tersebut,
39. Umur sani 7 tahun lebih tua dari umur Ari. Sedangkan
jumlah umum mereka adalah 43 tahun. Tentukanlah
Umur masing – masing.
40. Harga sebuah buku tulis dan sebuah buku gambar
Rp8.000,00. Sedangkan harga dua buku tulis dan
sebuah buku gambar Rp 11.000,00. Tentukan Model
matematika dari soal tersebut,
41. Harga sebuah buku tulis dan sebuah buku gambar
Rp8.000,00. Sedangkan harga dua buku tulis dan
sebuah buku gambar Rp 11.000,00. Tentukan Harga
satuan dari buku tulis dan buku gambar,
42. 4 pensil dan 2pena harganya Rp 7.400,00. 6 pensil dan
5 pena harganya 15.300,00, tentukan harga 1 buah
pensil dan 1 buah pena ?
43. Fatimah membelikan 3 buah baju dan 2 buah kaos
harganya 95.000, di toko yang sama amelia membeli 2
baju dan 4 kaos harganya 70.000. tentukan harga 2
buah baju dan 1 kaos?

44. Carilah persamaan garis jika diketahui letaknya


seperti pada gambar dibawah ini

Pengantar aljabar 123


y

45. Carilah persamaan garis jika diketahui letaknya


seperti pada gambar dibawah ini

46. Carilah persamaan garis jika diketahui letaknya


seperti pada gambar dibawah ini
y

Pengantar aljabar 124


47. Gambarlah persamaan grafik dari persamaan berikut
ini -3x + 2y = 6
48. Gambarlah persamaan grafik dari persamaan berikut
ini 2x + 3y = 12
49. Carilah koordinat titik potong antara dua garis
dibawah ini dan gambarkan grafiknya
Y = 2x – 4 dan y = x + 3
50. Carilah koordinat titik potong antara dua garis
dibawah ini dan gambarkan grafiknya
X +2y = -1 dan y = x + 5

Pengantar aljabar 125


PEMBAHASAN

1. 2x + y =8 y = 8 -2x
Y = 8 – 2x disubstitusikan kepersamaan x + y = 6
Maka, x + 8 – 2x = 6
X – 2x = 6 -8
-x = -2
X =2
Substitusikan x = 2 ke persamaan awal, 2x + y = 8
X = 2 diperoleh 2(2) + y = 8 y = 4
Jadi HP = (2,4)
2. X + 3y = 8 x1 x + 3y = 8
X + y = 2 _ x3 3x +3y = 6 _
2y = 6 -2x = 2
Y =3 x = -1
Jadi HP = (-1 , 3)
3. 5x + 4y = 24 x3 15x + 12y = 72
2x – 3y = 5 x4 8x – 12y = 20 _
23x = 92
X=4
X = 4 disubstitusikan ke persamaan 5x + 4y = 24
5(4) + 4y = 24
20 + 4y = 24
4y = 4
Y =1
Jadi HP = (4,1)
4. X + y = 7 x = -y +7
Substitusikan kedalam 2x – y = 2
2(-y + 7) – y = 2
-2y + 14 – y = 2
Pengantar aljabar 126
-3y = -12 Y=4
Substitusikan y = 4 kedalam x + y = 7
X+4=7 X=3
Jadi HP = (3,4)
5. 2x – 5y = 1 2x = 5y +1
X=
Substitusikan x = kedalam 4x – 3y = 9
4( ) – 3y = 9
10y + 2 – 3y = 9
7y = 7 y=1
Substitusikan y = 1 kedalam 2x – 5y = 1
2x – 5(1) = 1
2x =6
X=3
Jadi HP = (3,1)
6. X + y = 7 x2 2x + 2y = 14
2x – y = 2 _ x1 2x – y = 2 _
-x = 5 -3y = 12
X =-5 y= -4
Jadi HP = (-5 , -4)
7. 2x – 5y = 1 x2 4x – 10y = 2
4x – 3y = 9 x1 4x – 3y = 9 _
-7y = -7
Y=1
2x – 5y = 1 x3 6x – 15y = 3
4x – 3y = 9 x5 20x – 15y = 45 _
-14x = -42
X =3

Pengantar aljabar 127


Jadi HP = (3,1)
8. 4x + 3y = 12 x2 8x + 6y = 24
8x – 5y = 2 x1 8x – 5y = 2 _
11y = 22
Y=2
Substitusikan y = 2 kedalam 4x + 3y = 12
4x+ 3(2) = 12 4x = 6 x=
Jadi HP = ( , 2)
9. 3x – y = 2 y = 3x – 2
Substitusikan kedalam 2x + 3y = 16
2x + 3(3x – 2) = 16
2x + 9x – 6 = 16
11x = 22
X =2
Y = 3x – 2
Y = 3(2) – 2
Y=4
Jadi HP = (2,4)
10. 0
0_
5y = 10
Y =2
Substitusikan y = 2 kedalam 2x + y = 20
2x + 2 = 20 2x = 18
X=9

Nilai x – 2y adalah 9 – 2(2) = 5

11. 4x – y = 15 Y = 4x – 15

Pengantar aljabar 128


Substitusikan ke dalam 3x + 2y = 6
4x + 2(4x – 15) = 6
4x + 8x – 30 = 6
12x = 36 X =3
Y = 4x -15
Y = 4(3) – 15 y = -3 HP = (3, -3)
12.
0 0+
= 48 X
Subtitusikan kedalam

0 Hp{ 6, -0,5}

13. x + y = 4  x = 4 – y
x = 4 – y disubstitusikan pada x – 2y = - 2 akan
diperoleh :
x – 2y = - 2
=(4 – y ) – 2y = - 2
=4 – 3y = - 2
=-3y = -6
=y = =2
selanjutnya untuk y =2 disubstitusikan pada salah satu
persamaan, misalnya ke persamaan x + y = 4, maka
diperoleh :
x+y=4
x+2=4
x=4–2=2
Jadi, penyelesaianya adalah x = 2 dan y = 2

Pengantar aljabar 129


14. x + y = 4
x – 2y = - 2 _
3y = 6 y=2
x+y =4 x2 2x + 2y = 8
x – 2y = - 2 x1 x – 2y = -2 _
3x = 6 x= 2
Jadi, penyelesaianya adalah x = 2 dan y = 2
15. Misalkan : harga 1 kg beras =x
Harga1 kg minyak goreng = y
Maka dapat dituliskan :
.000
0. 00
Diperoleh model matematika :
.000
0. 00
16. .000
0.000
 Menentukan variabel x dari persamaan (1)
.000
.000 ...(3)
 Subsitusikan nilai x pada persamaan (3) ke persamaan
(2).
0.000
.000 0.000
.000 0.000
0.000 .000
000
.000...(4)
 Subsitusikan nilai y pada persamaan (4) ke persamaan
(2).
Pengantar aljabar 130
0.000
.000 0.000
0.000 .000
.000
.000
 Menentukan nilai x dan y.
Dari uraian tersebut diperoleh:
X = harga 1 kg gula =Rp4.000,00
Y = harga 1 kg kajang ijo = Rp 2.000,00
17.
_
-9y = -9 y=1
Substitusikan kedalam 4x – 2y = 2
4x – 2(1) = 2
4x = 4 x=1 HP =(1,1)
18.

substitusikan y = 3 kedalam

HP = (-2 , 3)
19. 0
+
9x = 36 X =4
Substitusikan kedalam 2x – 3y = 30
2(4) – 3y = 30
8 – 3y = 30
-3y = 22 y = -7,3
Pengantar aljabar 131
HP = (4 ; -7,3)
20.
+

Subsitusi kedalam x + 2y = 4
0
HP = (3,2 ; 0,4)
21.
+
0

Substitusikan kedalam 4x + 2y = 9

0
0
HP = (2,5 ; -1)
22. X + 3y = 8 x+y=2
Untuk x = 0 , y = 2,6 untuk x = 0 , y = 2
Y = 0 ,x = 8 y = 0 , x= 2
(8;2,6) (2,2)
X + 3y = 8 x1 x + 3y = 8
X + y = 2 _ x3 3x +3y = 6 _
2y = 6 -2x = 2
Y =3 x = -1
Jadi titik potongnya (-1 , 3)
23. 5x + 4y = 24 2x – 3y = 5
Untuk x = 0 , y = 6 untuk x = 0 , y = -1,6
Y = 0 , x = 4,8 y = 0 , x = 2,5

Pengantar aljabar 132


5x + 4y = 24 x3 15x + 12y = 72
2x – 3y = 5 x4 8x – 12y = 20 _
23x = 92
X=4
X = 4 disubstitusikan ke persamaan 5x + 4y = 24
5(4) + 4y = 24
20 + 4y = 24
4y = 4
Y =1
Jadi titik potongnya adalah (4,1)
24. x + y = 7 x = 7 – y
substitusikan kedalam 2x + y = 9
2(7 – y) + y = 9
14 – 2y + y = 9
-y = -5 y=5
Substitusikan kedalam x = 7 – y
X=7–5=2
y =5
HP = {2,5}
25. X – 2y = 5 x = 2y +5
Substitusikan 2x + 5y = 10
2(2y + 5) +5y = 10
4y + 10 +5y = 10
9y = 0
Y=0
Substitusikan X = 2y +5
X = 2(0) + 5 = 5 HP = (5,0)
26. x – y = - 1 x3 3x – 3y = -3
3x + 2y = - 13 x1 3x + 2y = - 13 _
-5y = 10
Pengantar aljabar 133
y = -2
substiitusikan kedalam x – y = -1
x + 2 = -1
x = -3
HP = (-3,-2)
27. x – 3y = 8 x3 3x – 9y = 24
3x - y = -8 x1 3x - y = -8 _
-8y = 32
y = -4
substitusikan kedalam x – 3y = 8
x + 12 = 8
x = -4
HP = (-4,-4)
28. 2x + y = 9 x 1 2x + y = 9
x + 2y = 3 x 2 2x + 4y = 6 _
-3y = 3 y = -1
Substitusikan kedalam x + 2y = 3
x + 2(-1) = 3
x =5
HP = (5,-1)
29. 3x – 2y = 6
4x + 2y = 22 +
7x = 28
x=4
substitusikan kedalam 3x – 2y = 6
3(4) – 2y = 6
12 – 2y = 6 -2y = -6 y = 3
HP = (4,3)
30. x + y = 1 x = -y + 1
substitusikan kedalam 2x + y = -2
Pengantar aljabar 134
2(-y +1) + y = -2
-2y + 2 + y = -2
-y = -4 y =4
Substitusikan kedalam x = -y +1 x = -3 HP = {-3,4}
31. X – y = -1 x = y – 1
Substitusikan kedalam x + y = 5
(y -1) + y = 5
2y = 6 y=3
Substitusikan kedalam x = y -1
X=3–1=2
HP = (2,3)
32. X + 4y = 3 x = -4y +3
Substitusikan kedalam 2x + y = 20
2(-4y + 3) + y = 20
-8y + 6 + y = 20
-7x = 14 x = -2
Substitusikan kedalam x = -4y +3
X = -4(-2) + 3
X= 8+3 = 11 HP = (11,-2)
33. 5x + y = 7 y = -5x + 7
Substitusikan kedalam 2x – 3y = 13
2x – 3(-5x +7) = 13
2x +15x – 21 = 13
17x = 34
X=2
Substitusikan kedalam y = -5x + 7
Y = -5(2) + 7
Y = -10 + 7 = -3
HP = (2, -3)
34. x + y = 6 2x – y = 0
Pengantar aljabar 135
untuk x = 0 y = 6 untuk x = 0 y =0
y=0x=6 y=0 x=0
x+y=6
2x – y = 0 +
3x = 6
x =2
substitusikan kedalam x + y = 6
2+y=6
Y=4
Jadi titik potongnya adalah (2, 4)
35. x + y = 10
x–y= 2 -
2y = 12
y=6
x + y = 10
x+6= 2
x = -4
HP = (-4, 6)
36. x – y = 1
x+y=7 -
-2y = -6
y=3
x+y=7
x+3=7
x =4
HP = (4, 3)
37. x – y = 6
2x + y = 18 +
3x = 24
x =8
Pengantar aljabar 136
x–y=6
8–y=6
-y = -2
y=2
38. Misalkan : umur Sani = x tahun
Umur Ari = y tahun
Diperoleh model matematika :

39. Dengan metode eliminasi, diperoleh:


 Menghitung variabel x

 Menentukan nilai x dan y


Dari uraian tersebut, diperoleh :
x = umur Sani = 25 tahun
Y = umur Ari = 18 tahun
40. Misalkan: harga buku tulis = x
harga buku gambar = y
dapat dituliskan :
.000
.000
41. X + y = 8000
2x + y = 11000 _
-x = -3000
X = 3000
X + y = 8000
3000 + y = 8000
Pengantar aljabar 137
Y = 5000
Jadi diperoleh harga satu buku 3000 dan satu buku
gambar 5000
42. 4x + 2y = 7.400 x5 20x + 10y = 37.000
6x + 5 y = 12.300 x1 x2 12x + 10y = 30.600
8x = 6.400
.
x = = 800

subsitusi
4x + 2y = 7.400
4 ( 800) + 2y = 7.400
3.200 + 2y = 7.400
2y = 7.400 – 3.200 = 4.200
.
y= = 2.100
jadi harga 1 buah pensil dan 1 buah pena adalah 800 +
2100 =2900
43. 3x + 2y = 95.000 x4 12x + 8y = 380.000
2x + 4y = 70.000 x2 4x + 8y = 140.000
8x =240.000
.
x = = 30.000
subsitusi
3x + 2y = 95.000
3(30.000) + 2y = 95.000
90.000 + 2y = 95.000
2y = 95.000 – 90.000 = 5.000
.
Y= = 2500
Harga 2 buah baju = 60.000
Jadi harga 2 buah baju dan 1 buah kaos seharga 62500

Pengantar aljabar 138


44. Dik : (-3 , 0) dan (0 , 5)
Dit : persamaan garis
Jawab : =

=
= 3y = 5x + 15
= 3y – 5x = 15
45. Dik : (-2 , 0) dan (0 , -4)
Dit : persamaan garis
Jawab : =

=
= 2y = -4x – 8
= 4x + 2y = -8
46. Dik : (2 , -1) dan (2 , -4)
Dit : persamaan garis
Jawab : =

= = - 3x + 6 = 0
47. y

Pengantar aljabar 139


48. Y

49. Y – 2x = – 4
Y– x= 3 _
-x = -7 x=7
Substitusikan x kedalam y – x = 3
Y–7=3
Y = 10
HP = (7, 10)

Pengantar aljabar 140


50. X +2y = -1
X–y=5 _
3y = -6
Y = -2
Substitusikan kedalam x – y = 5
X – (-2) =5
X=3
HP = (3 , -2)

Pengantar aljabar 141


BAB 8

SISTEM PERTIDAKSAMAAN

8.1 HIMPUNAN
8.2 NOTASI DAN ARTI PERTIDAKSAMAAN
1. SIFAT-SIFAT PERTIDAKSAMAAN
8.3 PERTIDAKSAMAAN KUADRAT
1. PENYELESAIAN MENGGUNAKAN SKETSA GRAFIK

Pengantar aljabar 142


1.1
8. HIMPUNAN
11.2

a. Pengertian himpunan

himpunan adalah segala koleksi benda-benda tertentu


yang dianggap sebagai satu kesatuan

b. Notasi dalam himpunan

himpunan ditulis menggunakan huruf besar, misalnya


S, A, atau B, sementara elemen himpunan ditulis
menggunakan huruf kecil (a, c, z). Cara penulisan ini
adalah yang umum dipakai, tetapi tidak membatasi bahwa
setiap himpunan harus ditulis dengan cara seperti itu.
Tabel di bawah ini menunjukkan format penulisan
himpunan yang umum dipakai.

Notasi Contoh
Himpunan Huruf besar
Elemen Huruf kecil (jika
himpunan merupakan huruf)
Kelas Huruf tulisan tangan

Himpunan-himpunan bilangan yang cukup dikenal,


seperti bilangan kompleks, riil, bulat, dan sebagainya,
menggunakan notasi yang khusus.

Pengantar aljabar 143


Bilangan Asli Bulat Rasional Riil Kompleks
Notasi

Simbol-simbol khusus yang dipakai dalam teori


himpunan adalah:

Simbol Arti
atau Himpunan kosong
Operasi gabungan dua himpunan
Operasi irisan dua himpunan
Subhimpunan, Subhimpunan sejati
, , ,
, Superhimpunan, Superhimpunan sejati
Komplemen
Himpunan kuasa

c. Macam-macam himpunan

1. Himpunan bilangan asli Himpunan bilangan asli adalah


himpunan bilangan yang anggota-anggotanya
merupakan bilangan bulat positif.

N = {1,2,3,4,5,6,......}

2. Himpunan bilangan prima Himpunan bilangan prima


adalah himpunan bilangan-bilangan asli yang hanya

Pengantar aljabar 144


dapat dibagi dirinya sendiri dan satu, kecuali angka 1.

P = {2,3,5,7,11,13,....}
3. Himpunan bilangan cacah

Himpunan bilangan cacah adalah himpunan bilangan


yang anggota-anggotanya merupakan bilangan bulat
positif digabung dengan nol.

C = {0,1,2,3,4,5,6,....}

4. Himpunan bilangan bulat

Himpunan bilangan bulat adalah himpunan bilangan


yang anggota-anggotanya seluruh bilangan bulat, baik
negatif, nol, dan positif.

B = {...,-3,-2,-1,0,1,2,3,...}

5. Himpunan bilangan rasional

Himpunan bilangan rasional adalah himpunan bilangan


yang anggota-anggonya merupakan bilangan yang
dapat dinyatakan sebagai:

p/q dimana p,q Î bulat dan q ¹ 0 atau dapat dinyatakan


sebagai suatu desimal berulang.

contoh: 0,-2, 2/7, 5, 2/11, dan lain lain

Pengantar aljabar 145


6. Himpunan bilangan irasional

Himpunan bilangan irasional adalah himpunan


bilangan yang anggota-anggotanya tidak dapat
dinyatakan sebagai sebagai p/q atau tidak dapat
dinyatakan sebagai suatu desimal berulang.
contoh: log 2, e, Ö7

7. Himpunan bilangan rii

Himpunan bilangan riil adalah himpunan yang


anggota-anggotanya merupakan gabungan dari
himpunan bilangan rasional dan irasional.
contoh: log 10, 5/8, -3, 0, 3

8. Himpunan bilangan imajiner

Himpunan bilangan imajiner adalah himpunan


bilangan yang anggota-anggotanya merupakan i
(satuan imajiner) dimana i merupakan lambang
bilangan baru yang bersifat i² = -1
contoh: i, 4i, 5i

9. Himpunan bilangan kompleks

Himpunan bilangan kompleks adalah himpunan


bilangan yang anggota-anggotanya (a + bi) dimana a, b
Î R, i² = -1, dengan a bagian riil dan b bagian imajiner.

contoh: 2-3i, 8+2

Pengantar aljabar 146


10. Himpunan Kosong

Himpunan kosong yaitu himpunan yang tidak


mempunyai satupun elemen atau himpunan dengan
kardinal = 0. Notasinya Ø atau { }.
Contoh:
P = { x|x adalah akar-akar persamaan persamaan
kuadrat x2 + 5x + 10 = 0 }, maka n(P) = 0

11. Himpunan Bagian

Himpunan A dikatakan himpunan bagian dari himpuan


B jika dan hanya jika setiap elemen A merupakan
elemen dari B. dalam hal ini B dikatakan superset dari
A. notasinya A B.
Contoh:
Misalkan A = { 1, 2, 3 } dan B = {1, 2, 3, 4, 5}, maka A
B.

12. Himpunan yang Sama

Himpunan A dikatakan sama dengan himpunan B, jika


dan hanya jika setiap elemen A merupakan elemen B
dan sebaliknya. Notasinya A = B <—> A B.
Contoh:
Jika A = { a, b, c } dan B = { c, a, b } maka A = B

Pengantar aljabar 147


13. Himpunan yang Ekivalen
Himpunan A dikatakan ekivalen dengan himpunan B
jika dan hanya jika cardinal dari kedua himpunan
tersebut sama. Notasinya A ~ B <—> n(A) = n(B)
Contoh:
Jika A = { 1, 2, 3,4 } dan B = { s, a, p, i } maka A ~ B
sebab n(A) = n(B) = 4

14. Himpunan Saling Lepas


Dua buah himpunan A dan B dikatakan saling lepas jika
keduanya tidak memiliki elemen yang sama. Notasi A
// B.
Contoh:
Jika A { x|x є P x b " {"

15. Himpunan Kuasa


Himpunan kuasa dari himpunan A ad

d. Menggambarkan himpunan

1. Diagram venn
Himpunan dapat digambarkan dengan diagram
venn. Dalam diagram ini himpunan semesta
digambarkan sebagai empat persegi panjang
sedangkan himpunan-himpunan di dalamnya
digambarkan sebagai lingkaran atau bentuk
geometri lain.
Anggota himpunan biasanya dinyatakan sebagai
titik
Contoh:
Pengantar aljabar 148
Dik S= {0,1,2,3,4,5,6,7,8,9,10,11,12,13,14}
A: {1,2,3,4,5,6} B:
{2,4,6,8,10} C: {3,6,9,12}
Dit: gambarlah diagram vennnya
Jawab:

2. Diagram garis (subset)

e. Operasi antar aljabar

1. Gabungan (union) notasi : ∪

Gabungan dari dua himpunan A dan B adalah himpunan


yang terdiri dari semua elemen yang menjadi anggota A
atau menjadi anggota B.

Pengantar aljabar 149


A ∪ B = {x / x ∈ A atau x ∈ B}
contoh:
A = {1,2,3}
B = {0,2,4}
Maka A ∪ B = {0,1,2,3,4}

2. Irisan (intersection) notasi : ∩

Irisan dari dua himpunan A dan B adalah himpunan yang


terdiri dari semua elemen persekutuan dari himpunan A
dan B.
A ∩ B = { x / x ∈ A dan x ∈ B}
contoh:
A={1,2,3,4}
B={3,4,5}
maka A ∩ B = {3,4}

3. Selisih notasi : -
Selisih antara dua himpunan A dan B adalah himpunan
yang terdiri dari semua anggota A yang bukan anggota
B.
A – B = { x / x ∈ A dan x B}

contoh
A={1234}
B={1,2}
A – B = {3,4}

Pengantar aljabar 150


8. NOTASI DAN ARTI PERTIDAKSAMAAN
2

Pertidaksamaan adalah kalimat yang mengandung


tanda atau notasi (lebih kecil), (lebih besar), (lebih
kecil atau sama dengan) atau (lebih besar atau sama
dengan).
Sebagai mana pada persamaan , penyelesaian dari
pertidaksamaan adalah nilai-nilai atau bilangan-bilangan yang
membuat kalimatnya menjadi benar
Contoh:
Berat bagasi seorang penumpang pesawat yang
diperbolehkan maksimum 20 kg. Jika kita misalkan bahwa x
kg adalah berat bagasi maka kita dapat menulis
pertidaksamaannya,
yaitu x 20

1. SIFAT-SIFAT PERTIDAKSAMAAN

1. Menambah bilangan atau bentuk aljabar kedalam


pertidaksamaan

Pertidak samaan dapat ditambah atau dikurang dengan suatu


bilangan atau bentuk aljabar. Penambahan tersebut tidak
mempengaruhi nilai atau tanda pertidaksamaan asal ruas-

Pengantar aljabar 151


ruas pada pertidaksamaan sama-sama ditambah atau sama-
sama dikurang

i. Jika a b maka a + c b +c
ii. Jika a b maka a + c b +c
iii. Jika a b maka a – c b – c
iv. Jika a b maka a – c b – c

Kegiatan
Gunakan bilangan-bilangan untuk menyelidiki sifat
di samping. Misal : 3 4 benar, apakah juga benar
3+1 4+1?

Sifat- sifat diatas dapat kita gunakan dalam menentukan


penyelesaian pertidaksamaan, seperti dijelaskan pada contoh
berikut.
Contoh
X–1 3
X – 1 +1 3 + 1 (ruas kiri dan ruas kanan ditambah 1)
X 4

2. Mengalikan bilangan terhadap pertidaksamaan

Jika suatu bilangan dikaitkan terhadap pertidaksamaaan


maka hasilnya tergantung kepada bilangan pengali tersebut,
jika pengali lebih besar nol maka tanda pertidaksamaan

Pengantar aljabar 152


tidak berubah, sedangkan jika pengali lebih kecil nol maka
tanda pertidaksamaan berubah atau dibalik

i. Jika a b dan c 0 maka ac bc


ii. Jika a b dan c 0 maka ac bc
iii. Jika a b dan c 0 maka ac bc
iv. Jika a b dan c 0 maka ac bc

Kegiatan gunakanlah bilangan-bilangan untuk menyelidiki


sifat di atas. Misalnya 3 4 benar, apakah juga benar 3 x 2
4 x 2?

Contoh :
Sifat-sifat diatas dapat digunakan untuk menentukan
penyelesaian pertidaksamaan berikut ini.
2x 3
2x 3
2x 3 (ruas kiri dan kanan dikali )
X
Jadi penyelesaiannya adalah X

8. PERTIDAKSAMAAN KUADRAT
3

Bentuk umum pertidaksamaan kuadrat mempunyai


bentuk umum ax2 +bx + c 0 dengan a ≠ 0 dan a, b, c
Pengantar aljabar 153
anggota bilangan real. Tanda yang digunakan pada
bentuk umum ini mewakili tanda-tanda pertidaksamaan
lain yaitu :
Contoh
X2 – 2x – 3 0
Penyelesaian pertidaksamaan kuadrat
Nilai x ynag memenuhi pertidaksamaan kuadrat ax2 + bx
+ c 0 disebut penyelesaian atau pemecahan
pertidaksamaan . penyelesaian pertidaksamaan kuadrat
dapat dicari dengan menggunkan garis bilangan.

Langkah-langkah untuk menentukan penyelesaian kuadrat


dengan garis bilangan adalah sebagai berikut:
i. Tentukan akar-akar dari bentuk ax2 + bx + c
ii. Tempatkan akar-akar yang diperoleh pada garis
bilangan, dan akar-akar tersebut selanjutnya
digunakan sebagai batas-batas interval
iii. Tetapkan tanda ( + ) atau ( - ) dari ax2 + bx + c
pada garis bilangan
iv. Tetapkan interval yang menjadi penyelesaian , yaitu
: yang bertanda ( - ) sebagai penyelesaian dari
pertidaksamaan ax 2 + bx + c 0 dan yang
bertanda ( + ) sebagai penyelesaian dari ax2 + bx +
c 0
Contoh
Tentukan penyelesaian dari a. x2 – 3x – 10 0
Jawab: . x2 – 3x – 10 = (x – 5) (x + 2) x1 = 5 x2 =
-2

Pengantar aljabar 154


Pada garis bilangan selang yang bernilai positif (+) adalah x
-2 atau x 5

Penggunaan diskriminan

Pada bagian terdahulu telah dibahas tentang


penggunaan diskriminan yaitu menyelidiki akar-akar
persamaan kuadrat . pada penyelidikan tersebut muncul
pertidaksamaan yang berbentuk linear . tentu saja
pertidaksamaan yang mungkin muncul dalam penyelidikan
seperti itu tidaklah hanya linear tetapi adakalanya merupakan
pertidaksamaan kuadrat, seperti yang dibahas berikut ini.
Contoh
Tentukan nilai p supaya akar-akar persamaan kuadrat x2 - px
+ p = 0 mempunyai akar-akar yang real dan berbeda.
Jawab
x2 - px + p = 0
D = (-p)2 – 4 1 p = p2 – 4p
Akar-akar real dan berbeda dengan D 0
p2 – 4p 0 p (p – 4) 0 p1 = 0 dan p2 = 4

Pengantar aljabar 155


Selang yang bertanda + pada garis bilangan adalah p 0 atau
p 4 , jadi nilai p yang memenuhi adalah 0 atau p 4

a. Menyelesaikan pertidaksamaan kuadrat dengan sketsa


grafik
Contoh
Carilah himpunan penyelesaian x2 – 5x + 6 0
Jawab
Buatlah seketsa grafik f(x) = y = x2 – 5x + 6 dalam hal ini,
a = 1 atau a 0 . oleh karena itu grafik dari f(x) = x2 – 5x
+ 6 akan terbuka keatas
Titik potong dengan sumbu x dieroleh bila y = 0
x2 – 5x + 6 = 0
(x -3) (x -2) = 0 x1 = 3 dan x2 = 2
Sketsa grafiknya:

F(x)

Dari grafik terlihat bahwa harga / nilai f(x) = x2 – 5x + 6


akan negatif pada selang tertutup (2,3) jadi, HP = { x / 2
x 3}

Pengantar aljabar 156


LATIHAN SOAL

1. Dari sekelompok anak terdapat 15 anak gemar bulu


tangkis, 20 anak gemar tenis meja, dan 12 anak gemar
keduanya. Jumlah anak dalam kelompok tersebut adalah
2. Dalam suatu kelas terdapat 47 siswa, setelah dicatat
terdapat 38 anak senang berolahraga, 36 anak senang
membaca, dan 5 orang anak tidak senang berolahraga
maupun membaca. Banyak anak yang senang berolahraga
dan senang membaca adalah
3. Diketahui: S = {a, b, c, d, e, f, g, h}, A = {a, b, c}, B = {c, d, e}.
maka komplemen (A ∪ B adalah
4. Jika P = {Tiga bilangan prima yang pertama}
Q = { bilangan asli kurang dari 10}
Maka Q P adalah
5. Jika himpunan semesta S ={1, 2, 3, 4, 5, 6, 7, 8, 9}, A = {1,
} dan B { } maka B’ A adalah
6. Diketahui:
A = {bilangan asli}
J = {bilangan ganjil}
P = {bilangan prima}
Dari ketiga himpunan di atas yang dapat menjadi
himpunan semesta bagi { } adalah
7. {-6, -4, -2, 0, 2} dinyatakan dengan notasi pembentuk
himpunan menjadi
8. A {x | x x prima genap}. A
9. A = {bilangan prima}, B = {bilangan ganjil}, C = {bilangan
genap}. Pernyataan berikut yang benar adalah

Pengantar aljabar 157


10. Dari sekelompok siswa yang terdiri dari 32 orang,
ternyata 18 orang suka makan bakso, 6 orang suka
akan bakso dan mie ayam ,dan 5 orang tidak suka
makan bakso maupun mie ayam. Banyak siswa yang
hanya makan mie ayam adalah

11. A = {bilangan prima kurang dari 7}


B = {faktor dari 105}
C = {tiga bilangan ganjil yang pertama}
D = {bilangan ganjil antara 1 dan 8}
Dari himpunan-himpunan di atas yang sama adalah
12. Diberikan A = {0, 1, 2, 3} dan B = {0, 2, 4, 6, 8}.
Maka A ∩ B
13. Jika P = {2, 3, 4, 5, 6, 7} dan Q = {1, 3, 5} maka Q – P =

14. Yang merupakan contoh himpunan kosong adalah


15. Jika n(A) = 15, n(B) = 24 dan n(A ∩ B) = 10 maka n(A
∪ B)
16. Diberikan P = {1, 3, 5, 7} dan Q = {0, 1, 2, 3, 4, 5}. Maka
P∪Q
17. Jumlah siswa di kelas IX ada 40 orang, 25 orang gemar
matematika, 18 orang gemar bahasa inggris. Maka
banyaknya siswa yang gemar keduanya adalah
18. H adalah himpunan semua bilangan asli n demikian

sehingga bentuk menghasilkan bilangan bulat


kurang dari 1, maka banyaknya himpunan bagian tak
kosong dari H adalah ....

Pengantar aljabar 158


19. Suatu fungsi f yang dirumuskan dengan f(x) = ax + b
diketahui bahwa f(1) = 3 dan f(–3) = 11. Nilai a dan b
berturut-turut adalah...
20. Nilai minimum dari fx=2x2+14x+24 adalah...
Nilai min = b2- 4ac-4a
= 142- 4.2.24-4.2
= 196-192-8
= 4-8 =-12
21. Suatu fungsi ditentukan f :x -5x-5. Bila dari fungsi itu
fa=30 maka nilai a adalah
22. Suatu fungsi kuadrat fx=x2+2x-3 dengan daerah asal
Grafik fungsinya adalah...
23. 2-5x+mx2-x-m Untuk harga- harga m yang manakah
fungsi ini dapat disederhanakan?
24. Jika f x x x maka nilai dari f(31)
adalah
25. Banyaknya himpunan bagian dari himpunan {y | (y2 –
4)(y2 – y 0 0} adalah
26. Pertidaksamaan untuk daerah yang diarsir pad garis
bilangan dibawah ini adalah
27. Pertidaksamaan x 8 dapat digambarkan dengan garis
bilangan yaitu
Gambarlah pertidaksamaan berikut ini
28. X 3
29. X 5
30. -3 X 7
31. Nilai x yang memenuhi pertidaksamaan | 3x + 2| > 5
adalah
32. Nilai x yang memenuhi pertidaksamaan | 2x – 3 | < 5
adalah
Pengantar aljabar 159
33. Jumlah harga P dan Q lebih murah daripada harga tiga
R, sedangkan harga P dikurangi harga Q lebih mahal
daripada harga R, maka pernyataan berikut yang benar
adalah
34. Dari pertidaksamaan a x–y dan b x y
a b diperoleh hubungan
35. Nilai x yang memenuhi pertidaksamaan 5 – x -4
adalah
36. Nilai x yang memenuhi pertidaksamaan – x
maka
37. Himpunan penyelesaian dari pertidaksamaan x2 – 3x –
0 0 adalah
38. Himpunan penyelesaian dari pertidaksamaan - x
10 adalah
39. Tentukan himpunan penyelesaian pertidaksamaan 2r –
untuk r peubah pada himpunan bilangan cacah
40. Tentukan himpunan penyelesaian pertidaksamaan 4t +
3 > 2t – 5, untuk t peubah pada himpunan bilangan
bulat
41. Himpunan penyelesaian untuk pertidaksamaan 5t – 19
t – 6, jika t peubahnya pada himpunan
bilangan cacah adalah
42. Himpunan penyelesaian dari 5x – x ∈ C adalah
43. Penyelesaian dari 2(x – x ∈ R adalah
44. Penyelesaian dari pertidaksamaan | | adalah

45. Tentukan himpunan penyelesaian pertidaksamaan 3s –


s untuk s peubah pada himpunan bilangan
asli
Pengantar aljabar 160
Tentukanlah penyelesaian dari
46. -2 x – 3 5
47. 3x – 4 2x + 5
48. -3x 9
49. X2 - 2x - 24 0
50. X2 – 5x + 6 0

Pengantar aljabar 161


PEMBAHASAN

1. n(A) = 15
n(B) = 20
n A∩B = 12
Ditanya : n ( S )
Jawab :
n(S) = n(A) + n(B) - n A∩B
= 15 + 20 – 12
= 23 .

2. n(S) =( n(O) + n(M) - n O∩M n X


47 = (38 + 36 - n O∩M
47 – 5 = 74 - n O∩M
42 = 74 - n O∩M
n O∩M = 74 – 42 = 32.

3. S = {a, b, c, d, e, f, g, h}
A = {a, b, c}
B = {c, d, e}
A ∪ B {f g h}

4. Jika P = {Tiga bilangan prima yang pertama}


Q = { bilangan asli kurang dari 10}
Maka Q P adalah
P = { 2, 3, 5}
Q = {1, 2, 3, 4, 5, 6, 7, 8, 9}
Q P { }

Pengantar aljabar 162


5. S ={1, 2, 3, 4, 5, 6, 7, 8, 9}
A = {1, 3, 5}
B = {2, 4, 6, 8}
B’ – A = {1, 3, 5, 7, 9} – {1, 3, 5} = {7, 9}.

6. karena A = { 0}
J = {1, 3, 5, 7, 9, 11, 13, 15, 17, 19}
P = {2, 3, 5, 7, 9, 11, 13, 15, 17, 19},
Jadi yang memuat semua angka [11, 13, 15, 17, 19}

7. itu merupakan bilangan genap yang di mulai dari -6


dan berakhir sampai 2.
8. Karena bilangan prima {2, 3, 5, 7, 9} dan {2}
merupakan bilangan prima genap.
9. A = {2, 3, 5, 7, 9}
B = {1, 3, 5, 7, 9}
C = {0, 2, 4, 6, 8}
A∩B { } A∪B { } A∩C
{ } dan A ∪ C {0 }.
Jadi yang benar A ∩ C { }.

10. n(S) = n(B) + n(BMA) + n(TS) + x


32 = 18 + 6 + 5 +x
32 = 23 + x
x = 32 – 23 = 9 orang.
11. Jadi, A = {2, 3, 5}
B = {3, 5, 7}
C = {1, 3, 5}
D = {3, 5, 7}

Pengantar aljabar 163


12. Jadi, A = {0, 1, 2, 3}
B = {0, 2, 4, 6, 8}
A ∩ B {0, 2}

13. Jadi, P = {2, 3, 4, 5, 6, 7}


Q = {1, 3, 5}
Q – P = {1}

14. Himpunan bilangan asli antara 5 dan 6.

15. Jadi, n(A ∪ B) = n A n B n A∩B


= 15 + 24 – 10 = 29

16. Jadi, P = {1, 3, 5, 7}


Q = {0, 1, 2, 3, 4, 5}
P ∪ Q {0 }

17. n(S) = n(M) + n(I) – n M ∩ I


40 = (25 – x) + (18 – x) – x
0 x
x = 43 – 40 = 3 orang

18. Bilangan asli mempunyai anggota himpunan mulai dari 1,


dst.
Sedangkan bilangan bulat mempunyai anggota himpunan
mulai dari 0 dst
Dalam soal ditanya bilangan bulat yang kurang dari,
sedangkan yang ditanyakan adalah bilangan asli dimana
anggotanya mulai dari dst.
Jadi, tdak mungkin mempunyai irisan atau irisannya nol.
Pengantar aljabar 164
19. f(x) = ax + b
f(1) = 3
f(-3)=11
Ditanya : Nilai a dan b
Jawab :
f(1) = 3 maka f(1) = a.1+b
3 =a+b (persamaan 1)
f(-3)=11 maka f(-3)= a.(-3)+b
11 = -3a+b (persamaan 2)
Eliminasi persamaan 1 dan 2
a +b =3
-3a + b = 11 -
4a = -8
a = -2 (Disubtitusi ke persamaan 1)
a+b=3
-2+ b = 3
b = 5.
20. 142- 4.2.24-4.2
= 196-192-8
= 4-8 =-12
21. f(a) = -5a – 5
30 = -5a – 5
30 + 5 = -5a
35 = -5a
a = -7.
22. Fx=x2+2x-3 ; D={x|- x x⋲R}
D = {-4, -3, -2, -1, 0, 1, 2}
F ( -4 ) = -42 + 2(-4) - 3 = 16 – 8 – 3 = 5
F ( -3 ) = -32 + 2(-3) – 3 = 9 – 6 – 3 = 0
F ( -2 ) = -22 + 2(-2) – 3 = 4 – 4 – 3 = -3
Pengantar aljabar 165
F ( -1 ) = -12 + 2(-1) – 3 = 1 – 2 – 3 = -4
F ( 0 ) = 0 + 0 – 3 = -3
F ( 1 ) = 12 + 2.1 – 3 = 1 + 2 – 3 = 0
F ( 2 ) = 22 + 2.2 – 3 = 4 + 4 – 3 = 5
Dari data di atas diperoleh nilai maksimum = 5, maka y
maks = 5
( Terdapat pada grafik D )
f (x) = x2+ 2x – 3
( x + 3 )( x – 1 )
x = -3 atau x = 1 ( Grafik melalui titik (-3,0)
dan (1,0 )
Jadi, grafik yang benar adalah grafik D karena melalui titik
(-3,0) dan (1,0).

23. x2-5x+m=0
x2-x-m = 0 +
2x2-6x=0
x2-3x=0
x(x-3) = 0
x=0 atau x=3
Untuk x = 0 maka x2-5x+m = 0
m=0
Untuk x = 3 maka x2-5x+m = 0
9 – 15 + m = 0
m=6
Jadi nilai m adalah 0 atau 6.
24. Diketahui f x x x dan ditanya f
maka
2x + 1 = 31
2x = 30
Pengantar aljabar 166
x = 15
maka x di subtitusi ke dalam persamaan x
12)(x + 13)
x x – 12 )(15 + 13)
= 3 x 28 = 84

25. (y2 – 4)(y2 – 7y + 10) = 0


(y – y 0 ∨ y – 5)(y – 2) = 0
y=±2 y=5 V y=2
n(y) = {-2, 2, 5} = 23 = 8
26.

Jawab :x 4

27.

28. X 3

Pengantar aljabar 167


29. X 5

30. -3 X 7

31. | 3x + 2| > 5
3x + 2 < -5 atau 3x + 2 > 5
3x < -7 atau 3x > 3
X < atau x x∈R
HP = {x | x < atau x x∈R

32. | 2x – 3 | < 5
x - atau x
2x < -2 atau 2x < 8
X< atau x x∈R
HP = {x | x < atau x x∈R

33.

Jadi, Q lebih murah daripada R

Pengantar aljabar 168


34. b x y a b pindah ruas jadi a b x– y b

35. 5 – x -4
x - kedua ruas ditambah -5)
- x -9
x

36. – x
8– –2– x – 2 (ditambahkan dengan -2)
- x
- x -5 (dikalikan dengan )

37. x2 – 3x – 0 0
x2 – 3x – 10 = 0
(x – 5)(x + 2) = 0
x– 0 ∧ x 0
x=5 x = -2
jadi, himpunan penyelesaiannya adalah - x .

38. - x 0

-6 . < . x 0. (kedua ruas dikalikan )


- x

39. 2r –
 2r – kedua ruas ditambah

Pengantar aljabar 169


 r
 2r . . (kedua ruas dikalikan )
 r
Jadi, himpunan penyelesaiannya adalah {0, 1, 2, 3, 4, 5,
6}.
40. 4t + 3 > 2t – 5
 4t + 3 – 3 > 2t – 5 – 3 (kedua ruas ditambah – 3)
 4t > 2t – 8
 4t – 2t > 2t – 2t – 8 (kedua ruas ditambah – 2t)
 (4 – 2)t > -8
 2t > -8
 2t . > -8 . (kedua ruas dikalikan )
 t > -4
Jadi, himpunan penyelesaiannya adalah {-3, -2, - 0 }.

41. 5t – t –6
5t – t –6
5t – t
5t – 2t –
3t –
t ruas kedua ditambah
t
. . (kedua ruas dikalikan )
t
Jadi, HP = {0, 1, 2, 3, 4, 5, 6, 7}

Pengantar aljabar 170


42. Himpunan penyelesaian dari 5x – x ∈ C adalah

Jawab :
5x –
5x – kedua ruas ditambah
x 0
x setelah kedua ruas dikalikan
jadi himpunan penyelesaiannya adalah { }

43. 2(x –
2x –
2x – kedua ruas ditambah
x
. x . (kedua ruas dikalikan )
x

44. | |
3x + 4 < 8 atau 3x + 4 > -8
3x < 4 3x > -12
X< x > -4
Jadi, himpunan penyelesaiannya adalah -4 < x <

45. 3s – s
 3s – s kedua ruas ditambah
 s s

46. 2 x – 3 5
2 + 3 x – 3 +3 5 +3
Pengantar aljabar 171
5 x 8
Jadi penyelesaiannya adalah 5 x 8
47. 3x – 4 2x + 5
Jawab : 3x – 4 2x + 5
3x – 2x 5 + 4
X 9
48. -3x 9
Jawab : -3x 9
-3x 9 (ruas kiri dan kanan dikali - )
X -3
49. X2 -
2x - 24 0
Jawab : X2 - 2x - 24 = (x – 6)(x + 4) x1 = 6 x2 = -4
Misal x = 0 02 – 2(0) – 24 = (-)
X = 7 72 – 2(7) – 24 = (+)
X = -5 (-5)2 – 2(-5) – 24 = (+)

Jadi penyelesaiannya { -4 x 6}

50. X2 – 5x + 6 0
Jawab : X2 – 5x + 6 = (x – 5)(x – 1) x1 = 5 x2 = 1
Misal x = 0 02 – 5(0) + 6 = (+)
X = 5 52 – 5(5) + 6 = (-)
X = 1 (1)2 – 5(1) +6 = (+)

Pengantar aljabar 172


Jadi penyelesaiannya { -4 x 6}

Pengantar aljabar 173


BAB 9

PERSAMAAN KUADRAT

9.1 DEFINISI BENTUK AKAR


9.2 OPERASI ALJABAR BENTUK AKAR
1. Menjumlah dan mengurang bentuk akar
2. Mengali dan membagi bentuk akar
9.3 MERASIONALKAN PENYEBUT

Pengantar aljabar 174


9.1.1 DEFINISI BENTUK AKAR
1

Tanda akar dinotasikan dengan “ √ ” akar disingkat


dari akar pangkat dua)merupakan kebalikan dari kuadrat .
pernyataan yang ditulis dengan tanda akar disebut bentuk
akar

Perhatikan bahwa

1. Penarikan akar √ menghasilkan bilangan real apabila


x 0
2. Nilai dari √ adalah positif untuk x 0 Untuk x = 0
3. Jika x 0 maka √ menghasilkan bilangan tidak
real(hayal)

9. OPERASI ALJABAR BENTUK AKAR


2
Operasi aljabar seperti tambah , kurang,kali, bagi dapat
dilakukan terhadap bentuk akar. Operasi tersebut selanjutnya
digunakan untuk merasonalkan penyebut yang dinyatakan
dalam bentuk akar

1. Menjumlah dan mengurang bentuk akar

Pengantar aljabar 175


Bentuk akar yang sejenis dapat ditambahkan atau
dikurangkan . caranya adalah menambahkan atau
mengurangkan bilangan yang didepan bentuk akar
tersebut
a√ + b √ = (a + b) √
a√ – b √ = (a – b) √
contoh
2√ + 4√ = (2+4) √
=6√

2. Mengali dan membagi bentuk akar

Bentuk akar yang sejenis dapat dikalikan atau dibagi.


Caranya adalah menjadikan bentuk tersebut menjadi
satu akar dan mengalikan atau membagi bilangan yang
di bawah akar
√ √ =√
√ √ =a
√ √ =√

√ √
=√
√ √

Contoh

√ √ =√ =√

Pengantar aljabar 176


9. MERASIONALKAN PENYEBUT
3
Pecahan yang penyebutnya mengandung bentuk akar ,
dapat diubah atau disederhanakan dengan merasionalkan
bentuk akar yang terdapat pada penyebut . untuk
merasionalkan penyebut tersebut maka pembilang dan
penyebut dari pecahan sama-sama dikali dengan bentuk
senama atau sekawan dari bentuk akar yang terdapat pada
penyebut. Perhatikan rumusan berikut.

√ √
a. = =
√ √ √
√ √
b. = =
√ √ √
√ √
c. = =
√ √ √
√ √ √ √
d. = =
√ √ √ √ √ √
√ √ √ √
e. = =
√ √ √ √ √ √

Contoh:

√ √
= =
√ √ √

Pengantar aljabar 177


LATIHAN SOAL

1. Bilangan dibawah ini yang bukan bilangan rasional


adalah
2. Bilangan berikut yang merupakan bilangan rasional
adalah
3. Berikut ini yang bukan bilangan irasional adalah..
4. Bilangn 1,2323232323 termasuk biangan
5. Bilangn 1,14213562373 termasuk biangan
6. Nilai dari √ sama dengan

7. √

8. Jika √ = i maka √ 00 dapat ditulis menjadi.


9. Jika x = 81 maka x adalah
2

10. Jika x2 = y maka x =


11. Tentukanlah nilai dari √
12. Tentukanlah nilai dari √
13. Tentukanlah nilai dari √
14. Bentuk √ sama dengan nilainya dengan
15. Nilai dari sama dengan
16. =
17. Nilai dari √ sama dengan
18. 2√ +3√ + 4√
19. √ + √ +√
20. √ +√ +√
21. √ +√ -√

Pengantar aljabar 178


22. √ √
23. √ √
24. √ √ √
25. √ : √
26. √ : √
27. √ : √ √
28. ( √ √ )(√ √ )
29. (√ √ )
30. 3√ √
31. 7√ √
32. 3√ √
33. √ √
34. √ √ √
35. √ √ √
36. 2√ (√ √ )
37. (√ + √
√ √
38.

39. (√ √
40. (√ √ (√ +√
41. (√ √
√ √ √
42.

43.


44.

45.

Pengantar aljabar 179


46.

47.
√ √
48. Karena 43 = 64 maka √
49. √
50. √

Pengantar aljabar 180


PEMBAHASAN

1. √
2. O,333333...
3. √
4. Rasional
5. Irasional
6. 7
7.
8. √ 00 = √ 0
= 10i
9. √ =√
X=9
10. X2 = y (diakarin)
√ =√
X =√
11. √ = √ = 2√
12. √ = √ = 3√
13. √ = √ = 5√
14. √ =
15. =√ =4
16. = = =

17. √ = = 22 = 4
18. 2√ +3√ + 4√ = 9√
19. √ + √ +√
= √ + 2√ + 3√ = 6√

Pengantar aljabar 181


20. √ +√ +√
=√ +√ +√
= 2√ + 3√ + 4√ = 9√
21. √ + √ - √
=√ +√ –√
= 4√ + 3√ - 9√ = 0
22. √ √
=√
23. √ √
=5
24. √ √ √
=√
=√ =6
25. √ : √
=√ =√ =2
26. √ : √
=√ =√
27. √ : √ √
=√ =√ =4
28. ( √ √ )(√ √ )
=( √ √ )(√ √ )
( √ √ ) ( √ √ ) (√ √ )
(√ √ )
√ 0 √ 0 0 √ 0
29. (√ √ ) √ 0 √ 0
30. 3√ √ = (3 + 5) √ √

Pengantar aljabar 182


31. 7√ √
= (7 – 3) √

32. 3√ √
= 3 . 2√ √
√ √

33. √ √
√ . √ .
√ √
= (4 + 16)√ = 16√
34. √ √ √
√ √
35. √ √ √
( . √ ) ( . √ ) √
√ √
36. 2√ (√ √ )
( √ √ ) ( √ √ )
√ √
37. (√ + √
= (√ + √ (√ + √
= 2 +2√ +3
= 5 + 2√
√ √
38.

=√ = √ =2

Pengantar aljabar 183


39. (√ √
= (√ √ (√ √
= 2 - 2√ +3
= 5 - 2√
40. (√ √ (√ +√
= 2 - 2√ + 2√ - 3 = -1
41. (√ √
= (√ √ (√ √
= 2 - 2√ 0 + 5
= 7 - 2√ 0
√ √ √
42.

√ √ √
=

√ √
= =0

43.


=
√ √
= √

44.

√ √
= = √
√ √
45.


=
√ √

= =2+√
46.

Pengantar aljabar 184



=
√ √

= = √
47.
√ √
√ √
=
√ √ √ √
√ √
= = √ √

48. Karena 43 = 64 maka √


=√
√ =4
49. √

=√ =3
50. √
=√
=√ =2

Pengantar aljabar 185


BAB 10

PERSAMAAN KUADRAT

10.1 BENTUK UMUM PERXAMAAN KUADRAT


10.2 AKAR-AKAR PERSAMAAN KUADRAT
10.3 CARA MENYELESAIKAN PERSAMAAN KUADRAT
10.4 GRAFIK PERSAMAAN KUADRAT
10.5

Pengantar aljabar 186


B
10. BENTUK UMUM PERXAMAAN KUADRAT
1

Persamaan kuadrat adalah suatu persamaan dari suatu


variabel yang mempunyai pangkat positif dengan pangkat
tertinggi dua (berderajat dua).

Misalnya: i. X2 + 3x – 4 = 0

ii. 3x2 – x = 0

iii. x2 – 4x = 0 dan lain-lain

secara umum persamaan kuadrat mempunyai bentuk ax2 + bx


+ c = 0 dengan a, b, dan c ∈ dan a≠0

dalam persamaan kuadrat : ax2 + bx + c = 0

x merupakan variabel

a merupakan koefisien x2

b maka merupakan koefisien x

c merupakan konstanta

contoh 1 : a. persamaan kuadrat 3x2 – 4x + 1

koefisien x2 = 3, koefisien x = -4 dan konstanta


=1

Pengantar aljabar 187


B
10. AKAR-AKAR PERSAMAAN KUADRAT
2

nilai variabel x yang memnuhi persamaan kuadrat ax2


+ bx + c = 0 , a≠0 disebut akar-akar persamaan kuadrat .
akar-akar sering juga disebut penyelesaian atau pemecahan ,
akar-akar persamaan kuadrat umumnya dinotasikan dengan
x1 dan x2

contoh 2 : misal diketahui persamaan kuadrat yaitu : x2 –


5x + 6= 0 sekarang kita selidiki apakah 3 dan 4 adalah akar-
akar persamaan kuadrat tersebut, untuk menyelidikinya, nilai
x yang diberikan disubstitusi ke dalam persamaan , seperti
berikut ini.

x2 – 5x + 6= 0 misal x = 3 x2 – 5x + 6= 0 misal x = 4

32 – 5(3) + 6 =0 42 – 5(4) + 6 = 0

9 – 15 + 6 = 0 (memenuhi) 16 – 20 + 6 ≠0 (tidak
memnuhi)

Penyelidikan diatas menunjukkan bahwa 3 memenuhi


persamaan dan 4 tidak memnuhi, dengan demikian 3 disebu
akar atau penyelesaian sedangkan 4 bukan penyelesaian.

Pengantar aljabar 188


B
10. CARA MENYELESAIKAN PERSAMAAN KUADRAT
3

Sebuah persamaan kuadrat mempunyai satu atau dua


akar. Akar-akar tersebut dapat dicari dengan menggunakan
teknik-teknik mencari akar persamaan kuadrat, seperti:
akan digunakan dapat disesuaikan dengan bentuk
persamaan kuadratnya.

1. Menentukan akar-akar persamaan kuadrat dengan


pemfaktoran
Jika akar-akar persamaan kuadrat ax2 + bx + c
= 0 adalah irasional maka bentuk persamaan
kuadratnya dapat dicari dengan memaktorkan , dalam
teknik memaktorkan secara umum persamaan kuadrat
ax2 + bx + c = 0 diubah menjadi (x – )(x – ) = 0
berikut ini adalah persamaan kuadrat yang mudah
difaktorkan.

Persamaan kuadrat ax2 + bx + c = 0 dapat dicari denagn


memaktorkan atau memisahkan x sesuai dengan sifat
distributif, yaitu x (ax + b) = 0

Tentukan akar- akar persamaan kuadrat berikut dengan


memaktorkan
a. X2 +4x = 0
X(x + 4) = 0
Jadi x1 = 0 x2=-4
a. Persamaan kuadrat x2 + bx + c = 0

Pengantar aljabar 189


Akar-akar persamaan kuadrat x2 + bx + c = 0, dapat
dicari dengan memfaktorkan secara mudah, jika akar-
akar persamaan kuadratnya adalah bilangan rasional.

Dalam memfaktorkan:

x2 + bx + c = 0 diubah menjadi (x + p) (x +q) dengan p


+ q = b dan pq= c

contoh
tentukan akar-akar persamaan kuadrat x2 + 4x + 12 =
0
? 12 -6 -12
? 2 +
-4 -4

x2 + 4x + 12 = 0 (x – 6) (x + 4)

x1=6 x2= -4

b. Persamaan kuadrat a x2 + bx + c = 0
Akar-akar persamaan kuadrat a x2 + bx + c = 0 , dapat
dicari dengan pemfaktoran secara mudah jika akar-
akarnya merupakan bilangan raional, dalam
pemfaktoran:
i. a x2 + bx + c = 0 dapat diubah menjadi :
(ax + p) (ax + q) = 0

ii. atau a x2 + bx + c = 0 dapat diubah menjadi :


(mx + r) (nx + s) = 0

Pengantar aljabar 190


dengn ketentuan mn = a ; rs = c dan ms + nr = b

contoh :

tentukan akar-akar persamaan kuadrat 2x2+ 3x – 14 =


0 dengan memfaktorkan

2x2+ 3x – 14 (2x – 4) (2x + 7) = 0 (x –


2)(2x+7)=0

= x1 = 2 x2 = -3,5

melengkapkan bentuk kuadrat sempurna

di dalam melengkapkan kuadrat , persamaan kuadrat


ax2 + bx + c = 0 diubah menjadi (x+p)2 = q
agar proses pengerjaan di dalam melengkapkan
kuadrat lebih mudah maka sebaiknya koefisien melalui
pemisahan x2 yaitu a benilai 1 atau dibuat bernilai 1.
Contoh:
(carilah akar-akar persamaan kuadrat x2 – 4x – 1 = 0
Jawab :
X2 – 4x – 1 = 0
X2 – 4x = 1

Pengantar aljabar 191


(X – 2)2 = 5
X–2= √
X=2 √
X1 = 2 + √ dan x2 = 2 √
2. menentukan persamaan kuadrat dengan rumus abc
nilai akar-akar persamaan kuadrat ax2 + bx + c = 0
dapat dirumuskan dengan rumus yang disebut rumus
abc , yaitu :

x1,2 =
contoh
tentukan akar-akar persamaan kuadrat berikut dengan
rumus abc 2x2 - 5x + 1 = 0
jawab : dik a = 2 b = -5 c =1
√ –
x1,2 =

=
√ √
X1 = dan x2 =

Pengantar aljabar 192


LATIHAN SOAL

1. akar-akar persamaan kuadrat x2 – 5x = 0 adalah


2. akar-akar persamaan kuadrat x2 + 6x = 0 adalah
3. persamaan 3x2 – 4x = 5x akar-akarnya adlah
4. akar-akar persamaan kuadrat x2 + 7x + 10 = 0 adalah
5. akar-akar persamaan kuadrat x2 + 6x – 16 = 0 adalah
6. akar-akar persamaan kuadrat x2 + 5x + 6 = 0 adalah
7. akar-akar persamaan kuadrat x2 – 4x = 5x – 14 adalah
8. akar-akar persamaan kuadrat x2 – 3x – 18 = 0 adalah
9. Akar-akar persamaan kuadrat x x – 3) = 0 adalah
10. Akar-akar persamaan kuadrat 0 0
adalah
11. Akar-akar persamaan kuadrat 0 0
adalah Tentukan nilai-nilai α dan β yang memenuhi:
12. α β dan αβ 0
13. α β dan αβ
14. α β 0 dan αβ 0
Selesaikan persamaan kuadrat berikut dengan
faktorisasi.
15. 0
16. 0
17. x
18. Persamaan grafik fungsi kuadrat yang mempunyai titik
balik minimum dan melalui titik adalah
19. Bila x1 dan x2 penyelesaian dari persamaan
. 0 dengan x1 > x2, maka nilai dari
2x1 + x2

Pengantar aljabar 193


20. Akar-akar persamaan 2log2 x – 6 . 2log x + 8 = 2log 1
adalah x1 dan x2. Nilai x1 + x2
21. Jika salah satu akar persamaan kuadrat : ax2 + 5x – 12a =
0 adalah maka a
22. Jika x1 dan x2 adalah akar-akar persamaan kuadrat x2 – 5x
+ (k + 3) = 0, dan x13 + x23 maka nilai k
23. Jika akar-akar persamaan x + 5x + a = 0 dua kali akar-
2

akar persamaan 2x2 + bx – 0 maka nilai a b


24. Fungsi f(x) = x2 – 4x + a mempunyai ekstrim -6. Fungsi
g(x) = ax2 – ax mempunyai jenis ekstrim
25. Jika ada dua harga m yang memenuhi agar garis y = mx +
1 menyinggung kurva y = x2 – 3x + 2, yaitu m1 dan m2
maka m1 + m2
26. Jika x1 dan x2 adalah akar-akar persamaan 3x2 + 5x – 2 =
0, maka nilai x12 + x22 adalah
27. Jika x1 dan x2 adalah akar-akar persamaan x2 + 2x – 5 = 0,
maka nilai adalah
.
28. Jika x1 dan x2 adalah akar-akar persamaan 2x2 x–3=
0, maka nilai x1 x2 + x1 x2 adalah
2

29. Persamaan kuadrat yang akar-akarnya - dan adalah


30. Persamaan kuadrat yang jumlah akar-akarnya -4 dan hasil
kali akar-akarnya adalah
31. Jika α dan β adalah akar-akar persamaan x2 + 2x – 15 = 0,
maka persamaan kuadrat yang akar-akarnya α – β–
adalah
32. Akar-akar dari persamaan 2x2 – 3x – 0 adalah α dan β.
Nilai α2 β 2

33. Jika x1 dan x2 adalah akar-akar persamaan kuadrat x2 – x


– 2 = 0 maka nilai dari + adalah

Pengantar aljabar 194


34. Akar-akar persamaan kuadrat x2 – 4x + 3 = 0 adalah
x1 dan x2. Persamaan kuadrat yang akar-akarnya 2x1
+ 5 dan 2x2 adalah
35. 3x + 4x – 5 = 0
2

36. x2 – 2x – 10 = 0
37.
38. p2 + q2
39.
40.
41. p3 + q3
42. akar-akar persamaan 2x2 - 2x – 1 = 0
43. akar-akar persamaan 3x2 - 6x + 1 = 0
44. akar-akar persamaan 6x2 - 2x – 1 = 0

Pengantar aljabar 195


PEMBAHASAN

1. x2 – 5x = 0
x(x – 5) = 0
x1 = 0 dan x2 = 5
2. x2 + 6x = 0
x(x + 6) = 0
x1 = 0 dan x2 = -6
3. 3x2 – 4x – 5x = 0
3x2 – 9x = 0
3x(x – 3) = 0
x1 = 0 dan x2 = 6
4. x2 + 7x + 10 = 0
= (x +5) (x + 2)
= x1 = -5 atau x2 = -2
5. x2 + 6x – 16 = 0
= (x +8) (x – 2)
= x1 = -5 atau x2 = 2
6. x2 + 5x + 6 = 0
= (x +3) (x + 2)
= x1 = -3 atau x2 = -2
7. x2 – 4x = 5x – 14
x2 – 9x + 14 = 0
= (x – 7) (x – 2)
= x1 = 7 atau x2 = 2
8. x2 – 3x – 18 = 0
= (x – 6) (x + 3)
= x1 = 6 atau x2 = -3
9. x x – 3) = 0
= x – 5 = 0 atau x – 3 = 0 (sifat perkalian)

Pengantar aljabar 196


= x 1= 5 atau x2 = 3
10. 0 0
= (x – 6)(x – 5) = 0
= x – 6 = 0 atau x – 5 = 0 (sifat perkalian)
= x1 = 6 atau =x2 = 5
11. 0 0
= (3x + 4)(x – 5) = 0
= 3x + 4 = 0 atau x – 5 = 0 (sifat perkalian)
= 3x = -4 atau x =5
= X1 = atau x2 = 5
12. α β dan αβ 0
α β dan αβ 0 α dan β
13. α β dan αβ
α β dan αβ α dan β
14. α β 0
dan αβ 0
α β 0 dan αβ 0  α 0 dan β 0
15. 0
=- b c 0. Kasus cari α β dengan α β
αβ 0
α dan β 0
0 0 (sifat perkalian)
 x = 0 atau 0
 x1 = 0 atau 2=4
16. 0
= 0
= 0 atau 0 (sifat perkalian)
= 6x = 1 atau 6x = 1
= X1 = atau x2 =

Pengantar aljabar 197


17.
a = 2, b = 3, c = - . Kasus Cari α β yang memenuhi
α β αβ -70
 α - dan β 0
  0
 ( )( )=0
 x1 = atau x2 =
18. Persamaan grafik fungsi kuadrat yang mempunyai titik
balik (p,q) dan melalui titik (x,y) adalah y = a(x – p)2 + q
Jika titik balik (1, 2) dan melalui (2, 3) maka
3 = a(2 – 1)2 + 2  3 = a + 2  a = 1
Jadi, persamaan grafiknya: y = (x – 1)2 + 2
Y = x2 -2x + 3
19. . 0
= . 0 misalkan 2x = a
= a2 – 12a + 32 = 0
= (a – 8)(a – 4) = 0
= a = 8 atau a = 4
= 2x = 8, x1 = 3
= 2x = 4, x2 = 2
= Jadi, 2x1 + x2 = 2 . 3 + 2 = 8

20. 2log2 x – 6 . 2log x + 8 = 2log 1,


misalkan 2log x = a  a2 – 6a + 8 = 0
 (a – 4)(a – 2) = 0
 a = 4 atau a = 2
2log x = 4  x1 = 24 = 16

2log x = 2  x2 = 22 = 4

Pengantar aljabar 198


Jadi, x1 + x2 = 16 + 4 = 20
21. ax2
+ 5x – 12a = 0
x = 0  4a + 10 – 12a = 0
10 – 8a = 0
10 = 8a (kedua ruas dibagi 2)
5 = 4a Jadi, 4a = 5

22. x2 – 5x + (k + 3) = 0
x13 + x23 = 35
(x1 + x2)3 – 3 x1x2(x1 + x2 )= 35
(5)3 – 3(k + 3)(5) = 35 (dibagi 5)
25 – 3k – 9 = 35
9 = 3k
3 =k
Jadi, nilai k = 3.
23. x2 + 5x + a = 0
2x2 + bx – 3 = 0
2( )2 + b( ) – 3 = 0 (dikalikan 2)
x2 + bx – 6 = 0
b= 5
a = -6
a + b = -6 + 5 = -1
24. f(x) = x2 – 4x + a
y.ekstrim = -6
24 = 16 – 4a
4a = -8
a = -2
g(x) = ax2 – 2ax + 1
y. ekstrim =

Pengantar aljabar 199


= = 3
.
Jadi, g(x) = ax2 – 2ax + 1 mempunyai jenis ekstrim
3.

25. y1 = y2
x2 – 3x + 2 = mx + 1
x2 – (3m)x + 1 = 0
(m + 3)2 – 4 . 1 . 1. = 0
m2 + 6m + 5 = 0
m1 + m2 = = -6
26. X1 + x2 = = =
X1 + x2 = =
x12 + x22 = (x1 + x2)2 – 2 . (x1 x2)
=( ) ( )
= =
27. X1 + x2 = = = -2
X1 . x2 = = = -5

28. X1 + x2 = =
X1 . x2 = = =-
x12 x2 + x1 x2 = x1 . x2(x1 + x2)
= - = -
29. (x – x1)(x – x2) = 0
(x – (-2))(x – 5) = 0

Pengantar aljabar 200


(x + 2)(x – 5) = 0
x2 – 5x + 2x – 10 = 0
x2 – 3x – 10 = 0
30. x1 + x2 = -4
x1 x2 = 5
x2 – (x1 + x2)x + x1 x2 = 0
x2 – (-4) x + 5 = 0
x2 + 4x + 5 = 0

31. x2 + 2x – 15 = 0
a = 1, b = 2, c = -15
α β
α β
x1 α – 3 dan x2 β – 3
x1 + x2 α– β – 3)
α β–6

32. 2x2 – 3x – 9 = 0
a = 2, b = -3, c = -9
α β
α β
α2 β2 α β 2 – α β
= ( )2 – 2 ( )
= + = =

33. x2 – x – 2 = 0
a = 1, b = -1, c = -2

Pengantar aljabar 201


x1 + x2 = = =1
x1 x2 = = = -1
+ = = = -1

34. x1 + x2 = 4
x1 x2 = 3
x2 – α β x α β 0
x2 –((2x1 + 5)+(2x2 + 5))x+(2x1+ 5)(2x2+ 5)= 0
x2 –(2(x1+x2)+ 10)x + 4x1x2 + 10(x1+x2)+25 = 0
x2 –(2(4) + 10)x + 4(3) + 10(4) + 25 = 0
x2 – 18x + 12 + 40 + 25 = 0
x2 – 18 x + 77 = 0

35. a = 3, b = 4, c = -5
b2 – 4ac = 42 – 4(3)(-5)
= 16 + 60 = 76

36. a = 1, b = -2, c = -10


b2 – 4ac = (-2)2 – 4(1)(-10)
= 4 + 40
= 44

37. 3x2 – 4x + 2 = 0  a = 3, b = -4, c = 2


p+q = = =
pq = =
=

Pengantar aljabar 202


= =2

38. 3x2 – 4x + 2 = 0  a = 3, b = -4, c = 2


p+q = = =
pq = =
p2 + q2 = (p + q)2 – 2pq
=( ) -2( )
= - =

39. 3x2 – 4x + 2 = 0  a = 3, b = -4, c = 2


p+q = = =
pq = =

= = =

40. 3x2 – 4x + 2 = 0  a = 3, b = -4, c = 2


p+q = = =
pq = =

Pengantar aljabar 203


41. 3x2 – 4x + 2 = 0  a = 3, b = -4, c = 2
p+q = = =
pq = =
p3 + q3 = (p + q)3 – 3p2q – 3 pq2
= (p + q)3 – 3pq(p + q)
=( ) ( )( )
= = -
42. 2x2 - 2x – 1 = 0
a = 2 b = -2 c = -1
√ –
x1,2 =
√ √ √
= = =

√ √
X1 = dan x2 =
43. 3x2 - 6x + 1 = 0
a = 3 b = -6 c = 1
√ –
x1,2 =
√ √ √
= = =

√ √
X1 = dan x2 =
44. 6x2 - 2x – 1 = 0
a = 6 b = -2 c = -1

Pengantar aljabar 204


√ –
x1,2 =
√ √ √
= = =

√ √
X1 = dan x2 =

Pengantar aljabar 205

Anda mungkin juga menyukai